preguntas infectologia

195
INFECTOLOGIA ENARM

Upload: estelarisimo

Post on 05-Feb-2016

127 views

Category:

Documents


3 download

DESCRIPTION

Banco de preguntas infectologia

TRANSCRIPT

Page 1: Preguntas Infectologia

INFECTOLOGIA

ENARM

Page 2: Preguntas Infectologia
Page 3: Preguntas Infectologia

MENINGITIS• Cual es la presión de apertura a la toma de LCR en meningitis bacteriana?• R = Normal <180 o elevada > 180, siendo común 200-500

• Cuales son los datos del LCR de la meningitis bacteriana aguda?I. Presión de abertura: Normal o > 180 siendo común observar valores entre 2 – 500 II. Apariencia: Leucocitos y bacterias turban el LCRIII. Glucorraquia: Concentración < 40 (NORMAL > 45)IV. Proteinorraquia: Valores arriba de 50 (NORMAL 40)

• Cual es el tratamiento de elección en los contactos de pacientes con enfermedad meningocococica?

• R = RIFAMPICINA

• Que constituye el síndrome de austrian?

• R = MENINGITIS, ENDOCARDITIS y NEUMONÍA por estreptococo pneumoniae en PACIENTE ALCOHÓLICOS

Page 4: Preguntas Infectologia

MENINGITIS• Tratamiento de elección para meningitis por NEUMOCOCO?• R = CEFTRIAXONA

• Tratamiento de elección para meningitis por S. AUREUS METICILINO SENSIBLE?• R = Nafcilina, DICLOXACILINA

• Tratamiento de elección para meningitis por S. AUREUS METICILINO RESISTENTE?• R = VANCOMICINA o TEICOPLANINA

• Tratamiento de elección para meningitis por LISTERIA?• R = AMPICILINA

• Tratamiento de elección para meningitis por H. INFLUENZA?• R = CEFTRIAXONA o cefotaxima

• Tratamiento de elección para meningitis por ANAEROBIOS?• R = METRONIDAZOL

• Cual es el tratamiento de elección en la meningitis CRIPTOCOCOCICA?• R = ANFOTERICINA B + 5-fluocitosina

Page 5: Preguntas Infectologia

MENINGITIS• Que medida se debe tomar en los contactos de un paciente con meningitis por

Neisseria Meningitidis?• R = RIFAMPICINA oral a contactos cercanos o CIPROFLOXACINO.

• - Two students from a university dormitory building have contracted meningitis due to Neisseria meningitides. Which of the following students in the dormitory are most likely to benefit from chemoprophylaxis?

• (A) everybody in the dormitory, with oral amoxicillin• (B) close contacts only, with oral amoxicillin• (C) everybody in the dormitory, with oral rifampin• (D) close contacts only, with oral rifampin• (E) everybody in the dormitory, with meningococcal vaccine

Page 6: Preguntas Infectologia

- Although only close contacts need chemoprophylaxis, it is sometimes given more widely than recommended because of community concern. Meningococcal vaccine is effective against serotype A and C, and will prevent late secondary infection in close contacts. Ciprofloxacin or ofloxacin are alternatives to rifampin

Page 7: Preguntas Infectologia

MENINGITIS• Característica de meningoencefalitis subaguda? • Meningoencefelatis por tuberculosis

• Agente etiológicos en meningitis en recién nacidos?• E.COLI (en el trabajo de parto) y estreptococos del grupo B (AGALACTIAE)

• Agente etiológico de meningitis en adultos?• 1.Streptococo pneumoniae 2.Neisseria Meningitidis 3.Streptococo del grupo B

• CASO CLÍNICO: Recién nacido, con fontanela abombadas, petequias en la cara anterior del tórax, esta en cunero y alado entro por deshidratación. Actualmente con fiebre, rigidez de nuca, Kerney y Brusinski, punción lumbar con diplococos GRAM NEGATIVOS en LCR

• Agente etiológico?: • Meningitis por Neisseria Meningitidis (MENINGOCOCO) • Tx: • PGSC a dosis altas, al personal involucrado: cipro o rifampicina

• Medida implantada por la OMS para detección rápida de MENINGITIDIS?• DETERMINACIÓN DE ANTÍGENOS CAPSULARES: Streptococo pneumone, Neisseria Meningitidis,

Hemophilus Influenza se detecta en 3 horas –aunque no los solicites-

Page 8: Preguntas Infectologia

MENINGITIS• Meningitis POR LISTERIA (BACILO GRAM +) esta relacionado con:• LECHE O DERIVADOS LACTEOS, CARNES MAL COCIDAS, INMUNODEPRIMIDOS y px de la

tercera edad…. Tx :AMPICILINA como primera opción

• Con respecto del Streptococo Pneumoniae caract de infección?:• Se da principalmente en pacientes de los 2 a los 20 años, px ALCOHÓLICOS, con OTITIS

PREVIA.

• Meningitis por Bacilos GRAM NEGATIVOS caract de infección?• Se presenta en diabéticos, cirróticos, alcohólicos px con IVU, px con cáncer y/o

inmunodeprimidos. El bacilo GRAM POSITIVO y entérico mas importante la listeria monocitogena, otro bacilo que no es entérico es la hemophilus Influenza (disminución en su incidencia actualmente)

• Agente etiológico de la Meningitis en un px que previamente se le realizo procedimiento neuroquirurgico?

• STAPHILOCOCOS AUREUS

• Citocinas proinflamatorias que producen daño a nivel SNC, en la barrera hematoencefalica, plexos coroides?

• IL 1 Y FNT que aumentan en las primeras 2 horas

Page 9: Preguntas Infectologia

MENINGITIS• Cantidad normal de proteínas en liquido cefalorraquídeos?

• Normal: 40 nos pueden poner 45

• La meningitis es el prototipo de los tres tipos de edema cerebral, cuales son?:1) EDEMA VASOGENICO (FNT: aumenta la permeabilidad dela Barrera

hematopencefalica), 2) EDEMA INTERSTICIAL (a nivel del espacio subaracnoideo hay Exudado, leucocitos

producen obstrucción del flujo LCR produciendo este tipo de edema), 3) EDEMA CITOTOXICO (degranulación de los neutrófilos libera metabolitos tóxicos)

• Como obtenemos la presión perfusión intracerebral?• Presión arterial media menos la presión intracerebral… una disminución de una presión

arterial media puede provocar isquemia e infarto cerebral (p/ej sepsis)

• Cuadro clínico de px con meningitis bacteriana?1) FIEBRE , CEFALEA Y RIGIDEZ DE NUCA (hasta en el 90% de los casos) leucocitosis,

alteración del edo. De la conciencia.2) Nausea , vomito, si tiene RASH EN LA CARA ANTERIOR DEL TORAX (casi seguro

MENINGOCOCCEMIA). Datos de hipertensión intracraneal (papiledema, pupilas hiporreactivas, postura de descerebración, tiene reflejo de cushing)

Page 10: Preguntas Infectologia

MENINGITIS• Laboratorio de LCR?• PUNCIÓN LUMBAR: 1.-PRESION NORMAL DEL LCR 180mm de Agua, 2.-POLIMORFONUCLEARES: menos

de 5 polimorfos 3.-GLUCOSA: es el 50% dela glucosa central del px entre 45 y 55 4.-PROTEINAS: 40mg/dl 5.-AGUA DE ROCA 6.-CLORUROS: 90 a 110 en LCR

• En quienes se realiza el cultivo en TINTA CHINA?• CRIPTOCOCO NEOFORMANS EN PX CON VIH

• Caso clinico• Px con rigidez de nuca, brusinski +, irritado Cefalea, fiebre, sin traumatismo craneoencefálico,

ANTECEDENTE DE NADAR EN UN MANANTIAL LANZÁNDOSE CLAVADOS, a la punción lumbar: aspecto turbio, presión:200mmhg glucosa:20mg/dl proteínas:85 mg/dl Tinción de GRAM fue NEGATIVA, el cultivo se realizo, que tipo de meningitis tiene este px:

• a.-M aseptica b.-M purulenta c.-M Tuberculosa (Agente etiológico: NAEGLERIA o gardenelas son amibas de vida silvestre) muy agresivas, con pronostico mortal,

• Tx?: • ANFOTERICINA B

• La meningitis Viral caract del LCR:• CELULARIDAD: NORMAL PROTEÍNAS: ligeramente AUMENTADAS ASPECTO: AGUA DE ROCA

GLUCOSA: NORMAL PRESIÓN: nl o ligeramente NORMAL CLORUROS: NORMALES

Page 11: Preguntas Infectologia

MENINGITIS• CASO CLÍNICO: Px con fiebre intensa (40), ingresa al servicio de urg, con trastorno

del edo. De conciencia, con 2 crisis convulsivas.. de ocupación CUIDADOR DE CABALLOS (antecedente de haber enterrado a su caballo y después fue atacado por aves de rapiña)

• LCR: Glucosa: 100 liquido transparente, presión: 185mm de agua, celularidad: linfocitos cloruros normales

• Cual es la impresión dx: • MENINGOENCEFALITIS VIRAL EQUINA VENEZOLANA

• CASO CLÍNICO: Niña Indígena procedente de AREA RURAL, se presenta al servicio de urg. Con 3 semanas de evolución con fiebre persistente, vomito, trastorno del estado de conciencia, ala exploración física presenta papiledema ligero, con afectación del 3er, 4to, 6to par craneal, con rigidez de nuca. LCR: ASPECTO XANTOCROMICO, Presión: 220mm de agua Glucosa: 30 Proteínas: de 75mg/dl , Celularidad por linfocitos CLORUROS: 70 Meningitis tuberculosa

• Tx de meningitis en Recién Nacidos?• AMPICILINA (e.coli)- CEFOTAXIMA (streptococo)

Page 12: Preguntas Infectologia

MENINGITIS• Tx de meningitis en niños de 1 a 3 meses?• AMPI + CEFO o CEFTRIAXONA + (valorar administración de DEXAMETASONA) para evitar

VENTRICULITIS

• Tx de meningits en px de 3 meses y menos de 50 años?• CEFOTAXIMA + VANCOMICINA o CEFTRIAXONA + VANCOMICINA (cubrir cocos gram +, Y

bacilos gram -)

• Tx de meningitis en un px mas de 50 años con antecedente de alcoholismo o tiene DM• AMPICILINA + VANCOMICINA + CEFALOSPORINA (tx p cocos gram + bacilos gram -)

• Px neutropenico, o px con meningitis con TRAUMATISMO CRANEOENCEFALICO, o que le hicieron un procedimiento neuroquirurgico que tx se le dara?

• Cubrir Gram - (ceftazidil), cubrir Gram + meticilino resistente (Vancomicina)

• px que en el cultivo presenta meningococo que tx se da? • PENICILINA G si es resistente: CEFTRIAXONA O CEFOTAXIMA y si es alérgico VANCOMICINA (2

gr x dia)

• Px que le hicieron punción lumbar y obtuvieron en el LCR un cultivo Gram – que tx:• Pseudomona aeroginosa: Ceftazidine(6gr x dia) Kliebsella, e coli: CEFOTAXIMA O

CEFTRIAZONA (4gr x dia)

• Tx para: Listeria: AMPICILINA(12 gr al dia) H. Influenza: CEFTRIAXONA o cefotaxima Anaerobios: METRONIDAZOL (2 grx dia)

Page 13: Preguntas Infectologia
Page 14: Preguntas Infectologia
Page 15: Preguntas Infectologia
Page 16: Preguntas Infectologia
Page 17: Preguntas Infectologia

FOD• Cual es la definición de FOD?• R = Elevación de la temperatura por arriba de 38 GRADOS en varias determinaciones durante

mas de 3 SEMANAS sin llegar al diagnostico DESPUÉS DE 1 SEMANA de estudio hospitalario

• Cual es la definición de FOD en paciente hospitalizados?• R = Paciente hospitalizado 24 HRS CON FIEBRE y 2 CULTIVOS NEGATIVOS después DE 2 DÍAS.

• Define FOD en paciente inmunodeficiente?• R = FIEBRE >38.3 en paciente con NEUTROPENIA < 500 durante > 3 DÍAS y cultivos negativos

después DE 2 DÍAS.

• Define FOD asociado a VIH?• R = Fiebre por encima de 38.3 GRADOS y que permanece sin diagnostico por MAS DE 3

SEMANAS

• Cuales son las principales causas de FOD?1. Infección por TB y CMV2. Neoplasia por linfoma o leucemia3. Idiopática si después de 6m de estudio no hay causa

Page 18: Preguntas Infectologia

FOD

• Que posibilidades diagnosticas te sugieren los estudios de BH en FOD?

1. Neutrofilia: Infecciones bacterianas2. Linfocitosis: Virus, micobacterias, Brucella, Salmonella, Rickettsia y

Leishmania3. Eosinofilia: Neoplasias, parasitosis y vasculitis.4. Leucopenia: Neoplasia hematológica o inmunosupresión.5. > VSG: Sepsis, polimialgia reumática y AR.

• Cual es el pronostico del FOD?• R = Bueno aunque no se identifique la causa

Page 19: Preguntas Infectologia

FOD• Cual es el mecanismo de acción por el cual las citocinas generan fiebre?• Se liberan por los monocitos llegan por la sangre al SNC a nivel del hipotálamo y

estimulan la síntesis de PROSTAGLANDINAS E2 Y LA PROTAGLANDINA S2 ALFA que a su vez estimula el centro termorregulador para producir fiebre…

• Mecanismo por el cual los corticoides disminuyen la fiebre?• Son inmunosupresores inhiben la fagocitosis entonces no hay liberación de

citocinas y x lo tanto no hay fiebre

• Que diferencia hay entre hipertermia y fiebre?• En la hipertermia hay un daño a nivel hipotalámico (gralmente fármacos:

haloperidol, anestésicos inhalados p ej) y provocar una reacción idiosincrática… en la hipertermia hay temperaturas de 40-41 grados y no esta mediada por citocinas

• Efecto perjudicial de la fiebre en el embarazo?• En los primeros meses puede provocar anencefalia..

Page 20: Preguntas Infectologia

FOD• Mecanismo por el cual el paciente de larga evolución padece de anemia?• Las CITOCINAS PROINFLAMATORIAS compite por la TRANSFERRINA

• Características del síndrome febril agudo?• Paciente con fiebre menos de dos semanas de evolución, o fiebre cuantificada con sintomatología

inespecífica, por lo general debida principalmente a enf. Biliares son auto limitadas

• Caract de la fiebre de origen a determinar?• Fiebre mayor de 3 semanas que no cede, con fiebre mayor de 38 grados

• Etiología de la fiebre de origen indeterminado en un px con vih?• Pneumocistis jirovecci, Micobacterium avium Intracelular, Linfomas

• Cáncer mas frecuente en los niños? • Leucemia

• Y en los adultos jóvenes? • Linfoma

• Características del síndrome de horner? • Paciente con cáncer de pulmón, con ptosis y miosis, facies hipocráticas, EL TUMOR SE ENCUENTRA

DEL LADO DE LA CARA DONDE NO ESTA SUDANDO.

Page 21: Preguntas Infectologia

SEPSIS• Como se diagnostica el síndrome de respuesta inflamatoria sistémica?• R = Cuando se presentan 2 o mas de los siguientes parámetros:1. FIEBRE corporal > 38 o hipotermia < 362. TAQUIPNEA con FR > 24 x minuto3. TAQUICARDIA > 90 x minuto4. LEUCOCITOSIS > 12,000 o leucopenia < 4,000 o 10% de bandas

• Como diagnosticas sepsis grave?• R = Sepsis asociada a disfunción de un órgano con los siguientes datos:1. Cardiovascular: TAS < 90 mm Hg que responde a fluidoterapia2. Respiratoria: Relación FiO2/PaO2 < 2003. Renal: Diuresis < 0.5 ML/KG/HR durante 1 hr a pesar de fluidoterapia4. Trombocitopenia: < 80,000 o su reducción a la mitad con respecto a la obtenida 72 hrs

antes5. Acidosis metabólica: Con PH < 7.306. Choque séptico: Hipotensión con TAS < 90 durante 1 hr y que no responde a fluidoterapia

necesitando vasopresores

Page 22: Preguntas Infectologia

SEPSIS

• En caso de bacteriemia o sepsis cuales son algunas características distintivas de los agentes patológicos?

A. Neisseria meningitidis: Se acompaña de PURPURA O PETEQUIASB. Rikettsia: Cuando aparecen LESIONES PETEQUIALES por mordida de

garrapata en zona endémicaC. Pseudomona aureoginosa: ECTIMA GANGRENOSOD. S. Aureus o S. Pyogenes: ERITRODERMIA GENERALIZADA.

• Cual es la complicación mas frecuente de la bacteriemia?• R = SIRPA caracterizado por HIPOXEMIA E INFILTRADOS PULMONARES

DIFUSOS.

Page 23: Preguntas Infectologia
Page 24: Preguntas Infectologia

FASCITIS NECROSANTE• Cual es la secuencia de eventos para que sea manifiesta la fascitis

necrosante?• R = Dolor – Fiebre – aumento de volumen local con eritema e

hiperestesia – epidermis se indura adquiriendo un color rojo/café donde se forman vesículas - piel se necrosa y desprende

• Cuales son las variables de presentación de la fascitis necrosante?A. GANGRENA DE FOURNIER: Afecta escroto, pene, perineo con la

probable extensión a muslos y abdomenB. GANGRENA SINERGICA DE MELENEY: Da como resultado por la

combinación de S. Aureus + anaerobios.

• Cual es el agente causal de la fascitis necrosante?• R = ESTREPTOCOCO GRUPO A

Page 25: Preguntas Infectologia
Page 26: Preguntas Infectologia
Page 27: Preguntas Infectologia

ABSCESO CEREBRAL• Cual es la causa mas común de absceso cerebral?• R = INFECCIONES CRÓNICAS de los oídos y senos paranasales, en pacientes

trasplantados es debido a hongos- Aspergillus

• Cual es el cuadro clínico del absceso cerebral?• R = Similar al de un tumor endocraneal mas síndrome febril

• Localización mas frecuente del absceso cerebral?• R = Frontal

• Cual es el manejo del absceso cerebral?• R = CRANEOTOMÍA, PUNCIÓN O ASPIRACIÓN + PENICILINA G + METRONIDAZOL

considerando que el agente puede ser estreptococo, neumococo o anaerobios mixtos.

Page 28: Preguntas Infectologia

ENCEFALITIS• Cual es el agente etiológico de la encefalitis de San Luis?• R = Mosquitos en EUA

• Cual es el agente etiológico de la encefalitis equina oriental?• R = Afecta a caballos y a veces a humanos, se desconoce agente

• Cual es el agente etiológico de la encefalitis de La Crosse?• R = ARDILLAS en el bosque.

• Cual es el agente etiológico de la encefalitis del Nilo Occidental?• R = Esta se manifiesta en dichas regiones epidemiológicas.

• Cual es el manejo de las encefalitis virales?• R = SOPORTE, no dar esteroides.

Page 29: Preguntas Infectologia

NEUROCISTICERCOSIS• Cual es el agente infeccioso en neurocisticercosis?• R = Tenia SOLIUM

• Cual es el cc de la neurocisticercosis?1) Forma activa: Sin evidencia del parasito con hallazgos en la TAC de

calcificaciones o hidrocefalia2) Parenquimatosa: Larvas en corteza y ganglios basales3) Forma subaracnoidea: Con fibrosis leptomeningea provocando

neuropatías por atrapamiento o hidrocefalia

• Cual es el método diagnostico de elección para le neurocisticercosis?• R = TAC de elección e IRM en casos dudosos.

• Cual es el manejo de la neurocisticercosis?• R = ALBENDAZOL 400 mg c/12 1 SEMANA, PRAZICUANTEL X 15 DÍAS. SE

RECOMIENDA EL USO DE PREDNISONA 1 DÍA ANTES DEL FÁRMACO Y CONTINUAR CON DISMINUCIÓN DE LA DOSIS 14 DÍAS DESPUÉS.

Page 30: Preguntas Infectologia

TETANOS

• Cual es el cuadro clínico del tétanos?• R = Comienza con ESPASMOS LEVES EN LA MANDÍBULA (trismo), el

cuello y la cara. La RIGIDEZ se desarrolla rápidamente en el TÓRAX, ESPALDA, MÚSCULOS ABDOMINALES y en ocasiones la laringe interfiriendo con la respiración. Los espasmos musculares son contracciones súbitas, fuertes y dolorosas.

• Cual es el tratamiento de elección para tétanos?• R = PENICILINA SÓDICA CRISTALINICA 20, 000 000. Se debe

administrar globulina inmunitaria antitetánica 5000 U/IM y una vez recuperado el paciente se da esquema completo.

Page 31: Preguntas Infectologia

CASO CLINICO• -A 22-year-old recent immigrant to the United States has never

been vaccinated for tetanus. He sustains a minor, but soil-contaminated, injury. Which of the following statements is correct?

• (A) tetanus usually develops within 2 weeks following exposure• (B) tetanus always develops within 4 hours following exposure in

patients who have not been previously immunized• (C) tetanus may develop many months or years following

exposure in susceptible individuals• (D) the usual incubation period for tetanus is 48 hours• (E) tetanus may be prevented with penicillin

Page 32: Preguntas Infectologia

-In tetanus, an acute onset is usual. The median onset is 7 days, and 90% present within 14 days of injury. The organism is an anaerobic, motile Gram-positive rod. It has the ability to survive for years in the form of spores, which are resistant to disinfectants and heat. Tetanus can occur in nonimmunized individuals, or those who have neglected their booster shots. Penicillin, or metronidazole, is used in treatment, but their efficacy is not clear

Page 33: Preguntas Infectologia

CARBUNO O ANTRAX• Cual es el agente etiológico del Carbunco o ANTRAX?• R = BACILLUS ANTHRACIS, bacilo GRAM +

• La forma cutánea de carbunco o ántrax que es la mas común como se manifiesta?• R = Desarrollo de PÁPULA EN ZONA DE INOCULACIÓN, RODEADA DE VESÍCULAS, LA ZONA CENTRAL SE ULCERA Y SECA

DESARROLLÁNDOSE ZONA DEPRIMIDA DE COLOR NEGRO, NO DOLOROSO. Se resuelve espontáneamente pero 20% puede ser fatal.

• Como se manifiesta la forma intestinal de carbunco o ántrax?• R = FIEBRE, DISNEA, CIANOSIS, desorientación y signos de septicemia. Evoluciona rápidamente a choque, coma y muerte.

• Como se manifiesta el carbunco o ántrax respiratorio y quienes lo PADECEN FRECEUNTEMENTE?• R = Se manifiesta en personas que MANIPULAN PIEL Y LANAS. Se produce FIEBRE CON TOS NO PRODUCTIVA y mal estar

general, con buena evolución a 2-3 días y a continuación SÚBITAMENTE PRESENTA DIFICULTAD RESPIRATORIA GRAVE, CIANOSIS Y SEPTICEMIA FATAL CON MUERTE EN <24 HRS.

• Como realizas el diagnostico de carbunco o ántrax?• R = Identificación del bacilo con TINCIÓN GRAM, PCR.

• Cual es el manejo de carbunco o ántrax?• R = CIPROFLOXACINO, levofloxacino o PENICILINA G.

• Como previenes el carbunco o ántrax?• R = PROTECCIÓN DE PIEL, MUCOSAS Y LA VACUNACIÓN

Page 34: Preguntas Infectologia
Page 35: Preguntas Infectologia

SALMONELOSIS• Cuantos antígenos tiene la salmonella typhi?• R = 3: H FLAGELAR, K CAPSULAR y O SOMÁTICO

• Cuales son los sitios frecuentes de infección secundaria por salmonella typhi?• R = Hígado, bazo, la medula ósea, PLACAS DE PEYER DEL ÍLEON TERMINAL y la vesícula biliar.

• Cual es el cuadro clínico característico de salmonella typhi?• R = ROSÉOLA TIFOIDICA que se caracteriza por MACULAS ERITEMATOSAS DE 2-4 MM QUE BLANQUEAN A LA

PRESIÓN, localizadas en la parte superior del abdomen y tórax anterior y que por lo general dura de 2-3 días.

• Cual es la complicación mas frecuente de la salmonella typhi?• R = La mas común es la HEMORRAGIA GI que resulta de la erosión de un vaso de la pared intestinal secundaria a la

necrosis de las placas de peyer en el íleon terminal

• Cual es el medio diagnostico mas sensible en la primer semana?• R = HEMOCULTIVO, después el mielocultivo.

• Semana en la que se presentan las complicaciones de fiebre tifoidea?• R = Fines de SEGUNDA Y TERCERA SEMANA

• Con que inoculo de S. Tiphy se desarrolla la enfermedad?• R = 100,000

Page 36: Preguntas Infectologia

SALMONELOSIS• A que semana aparece la roséola tifoidica, la cual aparece a la digito presión?• R = 2da semana

• Como dx a un portador crónico de salmonella?• R = COPROCULTIVOS + DURANTE 1ª

• En que semana los px se perforan o tienen hemorragias con choque hipovolemico?• R = 3ERA semana

• En que semanas en la fiebre tifoidea se realizan los cultivos específicos?• R = 1era HEMOCULTIVO, 2da MIELOCULTIVO y 3era COPROCULTIVO/urocultivo

• Cual es el cuadro clínico de la salmonelosis?

• R = Fiebre elevada 2-3 semanas, dolor abdominal, DIARREA EN SOPA DE CHICHARO, ROSÉOLA, DELIRIO, ESPLENOMEGALIA.

• Cual es la prueba de laboratorio confirmatoria de fiebre tifoidea?

• R = 1:640 , LEUCOPENIA y cuadro clínico característico

Page 37: Preguntas Infectologia

SALMONELOSIS• Cual es el sitio donde suele albergarse la salmonella?• R = Vesícula biliar

• Que se presenta en la primera semana de la salmonelosis?• R = FIEBRE, HIPERSENSIBILIDAD ABDOMINAL, TIFLITIS (CUANDO HAY DOLOR EN FOSA ILIACA

DERECHA POR INVASIÓN DE LA PLACA DE PEYER Y CREPITA AL TACTO)

• Que se presenta en la segunda semana en la fiebre tifoidea?• R = DIARREA, estreñimiento, delirio, EXANTEMA EN CARA ANTERIOR DE TÓRAX QUE SEDE A LA

DIGITO PRESIÓN. El sistema retículo-endotelial se hipertrofia con HEPATO/ESPLENOMEGALIA, hiperplasia de las PLACAS DE PEYER. Bazo e hígado desarrollan nódulos tifoideos.

• Cuales son las complicaciones habituales de la fiebre tifoidea en la 3era semana?• R = HEMORRAGIA Y PERFORACIÓN INTESTINAL, peritonitis con placa simple de abd con aire

libre. Orquitis, meningitis, nefritis.

• Cual es el tratamiento de la salmonella multiresistente o en lugares o zonas endémicas?• R = CIPROFLOXACINO, levofloxacino, ceftriaxona

Page 38: Preguntas Infectologia

SALMONELOSIS

• Cual es el mejor método diagnostico en un paciente que se automedico en fiebre tifoidea?

• R = MIELOCULTIVO por que el medicamento no penetra a esa zona • Tratamiento para fiebre tifoidea?• R = CLORAMFENICOL 50/mg/kg por 2 SEMANAS

• Tratamiento para anemia + fiebre tifoidea?• R = AMOXICILINA o ampicilina, NO DARLE CLORAMFENICOL por ke

causa aplasia medular.

• Tx fiebre tifoidea en embarazada?• R = AMOXICILINA o ampicilina

Page 39: Preguntas Infectologia

SALMONELOSIS

• Tx fiebre tifoidea en niño con resistencia?• R = QUINOLONAS NO, cefotaxima o CEFTRIAXONA

• En que casos se utiliza la dexametasona en fiebre tifoidea?• R = En el ESTADO TIFOIDICO CON SHOQUE

• Que se le da al portador de salmonella?• R = CIPROFLOXACINA por 3 MESES

• Que tx das a un px portador crónico que no respondió a ciprofloxacino?

• R = COLECISTECTOMÍA, por que ahí vive la salmonella.

Page 40: Preguntas Infectologia
Page 41: Preguntas Infectologia

BRUCELOSIS• Cuales son los principales vectores de la brucelosis y a quienes afecta

comúnmente?• R= ORDEÑADORES, rastros, CARNICERO, veterinarios son vectores. DERIVADOS DE

LA LECHE como el queso o tejidos del animal afectado.

• Cuales son las cepas de brucella que produce enf en humano/ FIEBRE ONDULANTE?

• R = MELITERSIS DE CABRA + frecuente a nivel mundial, SUIZ DEL CERDO +, ABORTUS DE GANADO VOVINO +++.

• Microbiologicamente que es la brucella?• R = COCOBACILO GRAM -, crece a 37 grados, es inmóvil a pesar de TENER FLAGELO.

• Cual es la prueba de laboratorio para brucelosis?

• R = 2- MERCAPTOETANOL, es una inmunoglubulina IgG Vs BRUCELLA

Page 42: Preguntas Infectologia

BRUCELOSIS• Cual es el medio de cultivo de elección para brucelosis?• R = RUIZ CASTAÑEDA medio doble y ROSA DE BENGALA (fines epidemiológicos)

• Que tratamiento utilizas de primera elección contra brucella?• R = Combinado DOXICICLINA + GENTAMICINA de 3-6 semanas.

• Que tx utilizas en brucelosis en hueso y SNC?• R = DOXICICLINA Y RIFAMPICINA

• Cual es el tratamiento de brucelosis en embarazada?• R = TMP/SMZ+ RIFAMPICINA + ACIDO FÓLICO

• Cual es el tratamiento de niños menores de 12ª con brucelosis?• R = TMP/SMZ + RIFAMPICINA

• Cual es el tratamiento en pacientes alérgicos a sulfas en brucelosis?• R = RIFAMPICINA Y CEFTRIAXONA.

Page 43: Preguntas Infectologia
Page 44: Preguntas Infectologia

BOTULISMO

• Cual es el cuadro clínico del CLOSTRIDIUM BOTULINUM transmitido por alimentos?

• R = Los síntomas comienzan en 6 HRS DESPUÉS A 2 SEMANAS, se manifiesta con DIPLOPÍA, VISIÓN BORROSA, PTOSIS, disfagia, sensación de sequedad de mucosa oral, DEBILIDAD MUSCULAR que afecta únicamente a los hombros, miembros torácicos y mas tarde a los pélvicos.

• Cual es el manejo?• R = Administración de TOXINA BOTULÍNICA con PREVENCIÓN DE

INSUFICIENCIA RESPIRATORIA que amerite ventilación mecánica

Page 45: Preguntas Infectologia

CASO CLINICO

• - An 18-year-old woman has eaten homemade preserves. Eighteen hours later, she develops diplopia, dysarthria, and dysphagia. Which of the following is the most likely causative organism?

• (A) Clostridium botulinum toxin• (B) staphylococcal toxin• (C) salmonellosis• (D) brucellosis• (E) shigellosis

Page 46: Preguntas Infectologia

PESTE BUBONICA• Cual es el agente causal por la peste bubónica?• R = YERSINIA PESTIS

• Cual es el cuadro clínico de la peste bubónica?• R = Es la forma mas frecuente, se transmite por la PICADURA DE PULGA. Inicio con FIEBRE ALTA, nauseas,

MIALGIAS y ADENOPATÍAS DOLOROSAS REGIONALES, el PACIENTE EVOLUCIONA A ESTUPOR, COMA Y MUERTE.

• Que formas clínicas tiene la peste?• R = Peste bubónica, septicémica y neumónica

• Como diagnosticas peste?• R = BIOPSIA DE GANGLIOS o EXUDADO FARÍNGEO

• Cual es el tratamiento de la peste?

• R = ESTREPTOMICINA, CLORANFENICOL O TETRACICLINAS.

• Cual es la prevención de la peste?• R = Hay una vacuna con CEPA INACTIVADA F1

Page 47: Preguntas Infectologia

NAEGLERIA

• Cual es el agente causal de la enfermedad transmitida en los manantiales?

• R = Naegleria

• Cuale es el tratamiento de naegleria?• R = Anfotericina B

• Cual es el pronostico de naegleria?• R = Muerte

Page 48: Preguntas Infectologia

RABIA• Cual es el agente causal de la rabia?• R = Virus de la familia Rabdovirus

• Cual es el cuadro clínico de la rabia?• R = Dolor en el sitio de la mordedura, fiebre, malestar general, nausea y vomito. Diez días mas

tarde hay manifestaciones del SNC pudiendo ser encefálica o paralitica y las 2 formas evolucionan a coma - muerte.

A. ENCEFALICA: Se caracteriza por delirio, somnolencia, hidrofobia (espasmos laríngeos dolorosos al beber agua)

B. PARALITICA: Da una parálisis ascendente.

• Cual es el manejo de la rabia?I. Es necesario observar al animal los siguientes 10 DÍAS tomando UNA BIOPSIA DEL CUERO

CABELLUDO y analizarla con Ac fluorescente o PCR.II. Atención de la LESIÓN LAVANDO CON AGUA Y JABÓN ABUNDANTE A CHORRO DURANTE 10

MINUTOS, para MUCOSA ORAL O NASAL CON SOLUCIÓN FISIOLÓGICA DURANTE 5 MINUTOS. Desinfectar la herida con agua oxigenada o tintura de yodo.

III. Si se requiere SUTURAR LA HERIDA se debe APLICAR PRIMERO INMUNOGLOBULINA ANTIRRÁBICA HUMANA y se aproximan los bordes o se dan puntos temporales en caso que la herida sea profunda,

IV. En exposición leve se administra vacunación antirrábica en la región deltoidea los días 0-3-7-14 Y 28.

V. Se APLICARA INMUNOGLOBULINA INTRALESIONAL LA PRIMER ½ Y LA OTRA MITAD VÍA IM.VI. Se usara SUERO HETEROLOGO EN CASO EXTREMO DE NO CONTAR CON INMUNOGLOBULINA.

Page 49: Preguntas Infectologia

ADENOVIRUS

• Donde se replica el adenovirus?• R = Faringe, conjuntivas, intestino delgado, ganglios

linfáticos cervicales, preauriculares o mesentéricos.

• Cual es el cuadro clínico de adenovirus?• R = CONJUNTIVITIS aguda, QUERATOCONJUNTIVITIS,

FIEBRE CONJUNTIVAL, FARINGITIS LARINGOTRAQUEITIS (CRUP), bronquiolitis, neumonía, gastroenteritis, linfadenitis mesentérica y CISTITIS HEMORRÁGICA.

Page 50: Preguntas Infectologia

INFLUENZA

Page 51: Preguntas Infectologia
Page 52: Preguntas Infectologia

INFLUENZA

• Cuales son las proteínas que distinguen al virus de la influenza?

• R = H: HEMAGLUTININA y N: NEURAMINIDASA

• La influenza aviar que nomenclatura viral la representa?

• R = H5N1 y es altamente mortal

Page 53: Preguntas Infectologia

DENGUE Y FIEBRE AMARILLA• Cual es el mosquito transmisor del dengue?• R = AEDES AEPYPTI.

• Cual es el cc del dengue?1) El DENGUE CLÁSICO se inicia con FIEBRE, BRADICARDIA, pulso lento, EXANTEMA MACULAR

PUNTIFORME, tos, ardor faríngeo, MIALGIAS, ARTRALGIAS. 2) El DENGUE HEMORRÁGICO se caracteriza por aumento en la permeabilidad vascular con

clasificación del I-IV.

• Que es lo que distingue a la fiebre amarilla del dengue laboratorialmente?• R = CUERPOS DE INCLUSIÓN VIRAL llamados CUERPOS DE COUNCILMAN EN LA SANGRE.

• Cual es el cuadro clínico de la fiebre amarilla?• R = FIEBRE, cefalea, mialgias, DOLOR LUMBOSACRO, ERITEMA OCULAR Y FACIAL, PETEQUIAS EN

ENCIAS Y MUCOSA NASAL, HEMATEMESIS, MELENA, ALBUMINURIA, encefalitis o meningoencefalitis.

• Cual es el manejo de la fiebre amarilla y del dengue?• R = SINTOMÁTICO

Page 54: Preguntas Infectologia
Page 55: Preguntas Infectologia

VIH• Que células ataca el VIH?• R = CD4

• Cual es el ciclo vital del VIH?• R = A través de la proteína gp41, se realiza la fusión, posteriormente la RNA del VIH se descubre e

interna en la célula afectada, la enzima transcriptasa inversa del virion cataliza la transcripción inversa del RNA en DNA, este se transfiere hacia el núcleo en el cual se integra en los cromosomas por medio de la integrasa. Los macrófagos actúan como reservorio del VIH y lo diseminan a otros sistemas

• Que se debe realizar como prevención en una persona que halla sido puncionada de manera accidental con una aguja utilizada previamente en un paciente con VIH?

• R = INHIBIDORES NUCLEOSIDOS DE LA TRANSCRIPTASA REVERSA

• Como se manifiesta el síndrome retroviral agudo SRA en VIH?• R = SÍNTOMAS SIMILARES AL RESFRIADO O MONONUCLEOSIS, fiebre, escalofríos, sudores nocturnos

y erupciones en la piel que dura de 1-3 SEMANAS.

• De acuerdo a la CDC como se realiza dx de SIDA?• R = Recuento <200/MM DE CD4 + la presencia de ENFERMEDAD OPORTUNISTA

Page 56: Preguntas Infectologia

VIH• Como dx VIH?• R = ELISA y confirmatorio con WESTERN BLOOD

• Cuales son parte del grupo de fármacos que inhiben la TRANSCRIPTASA INVERSA en VIH?• R = ZIDOVUDINA, didanosina, zalcitabina.

• Cuales son las indicaciones actuales para uso de tratamiento retro vírico en VIH?1. Síndrome de infección aguda2. INFECCIÓN CRÓNICA: Enfermedad sintomática o asintomática con T CD4 <350/NL o RNA DEL

VIH CON > 50, 000 COPIAS3. PROFILAXIS POST- EXPOSICIÓN: Se recomienda utilizar combinaciones como ZIDOVUDINA +

LAMIVUDINA + NEVERIPINA

• Cuales son las indicaciones para cambiar el tratamiento antirretrovirico?1. Disminución <1 log del RNA del VIH con plasma a las 4 semanas de haber iniciado el

tratamiento2. Incremento DEL TRIPLE O MAS DEL VALOR PLASMÁTICO DEL RNA NO ATRIBUIBLE A INFECCIÓN

concomitantes o vacuolas.3. DISMINUCIÓN DEL PORCENTAJE DE CÉLULAS TCD44. Deterioro clínico5. Efectos adversos

Page 57: Preguntas Infectologia

VIH• Como se manifiesta la neumonía por Pneumocystis Jiroveci?

• R = CD4 < 200/MM mas AUMENTO DE LA DHL, el diagnostico definitivo se obtiene por la TINCIÓN DE ESPUTO CON

WRIGHT-GIEMSA

• Cual es la causa mas frecuente de retinitis en pacientes con VIH?• R = CITOMEGALOVIRUS

• Mecanismo por el que el VIH penetra a la célula?• R = Pinocitosis

• Cual es la neoplasia maligna mas común en pacientes con VIH?• R = Sarcoma de Kaposi

• Cual es el manejo del Sarcoma de Kaposi?

• R = Para la FORMA CUTÁNEA SE DA DOXORRUBICINA, para el INTESTINAL O VISCERAL SE DA

DOXORRUBICINA, BLEOMICINA Y VINBLASTINA.

• Como se maneja la profilaxis en pacientes con VIH de acuerdo al conteo de las células CD4?

1) CD4 < 200: VS P. JIROVECI con TMP/SMZ 1Tab/24 hrs C/3 DIA

2) CD4 75-100: VS M. AVIUM con CLARITROMICINA y se SUSPENDE CUANDO AUMENTAN LOS CD4

3) CD4 < 50: VS CITOMEGALOVIRUS con GANCICLOVIR4) PPD con induración > 5 MM iniciar profilaxis VS M. TUBERCULOSIS con ISONIACIDA a 300 mg al día +

PIRIDOXINA durante 9-12 MESES.

Page 58: Preguntas Infectologia

VIH• Tratamiento de primera elección para la retinitis por CITOMEGALOVIRUS?• GANCICLOVIR

• Tratamiento para el herpes tipo 2 en pacientes con VIH?• ACICLOVIR

• De que manera se puede inhibir el síndrome de desgaste?• TALIDOMIDA

• Agente infeccioso de la lengua vellosa?

• Virus de EPSTEIN BARR ( relacionada con virus de mononucleosis y linfomas) VIRUS TIPO 4

• Sitio mas común de lesión dermatológica, “SARCOMA DE KAPOSI” manchas color de piel?• El sitio de afectación mas frecuente es la CARA EN PARPADOS Y PUENTE NASAL RELACIONADO CON EL

VIRUS TIPO 8

• Tratamiento de toxoplasmosis?• PIREMITAMINA o sulfas + CLINDAMICINA en dosis altas

Page 59: Preguntas Infectologia

VIH• Posibilidad de contagio por coito anal receptivo en el VIH?• 1:100 1:30

• Zona donde se alojan el virus de la inmunodeficiencia es alta y útil para seguimiento del tx de VIH?• Biopsia de mucosa rectal otra opción es PCR

• Transmisión perinatal del VIH?: • 13% al 40%

• Coito con inserción anal: 1: 1,000• Transfusión: 1: 100,000• Coito inserción vaginal: 1: 10,000 varia si la mujer se encuentra en periodo menstrual o si tiene una

ETS

• Aguja infectada?: • 1-300

• Compartir agujas?: • 1-150 (drogadictos)

• A que se refiere la regla de los 3 con relación a aguja infectada?• Hepatitis B : 30% Hepatitis C: 3% HIV: .3%

Page 60: Preguntas Infectologia

VIH• Diagnostico para valorar el tratamiento “respuesta terapéutica”• RCP se realiza al mes y medio y a los 3 meses y posteriormente CADA 6 MESES (no se usa para diagnostico)

• Que pasa con una persona que tiene VIH y las siguientes enfermedades Concomitantes?• Hepatitis B: Indiferente (no pasa nada)• Hepatitis C: Agresividad• HEPATITIS G: FRENA LA PROGRESIÓN DEL VIH

• Con respecto a la estatificación de etapas A-B-C y grados I al III?• A: asintomáticos B: Síntomas Constitucionales (fiebre, perdida de peso, diarrea) C: Infecciones Oportunistas

(neumocistis jirovecci, linfoma del SNC, Criptosporidiasis, Toxoplasma del SNC, Micovacterium Avium intra celular)• I.- 500 copias II.-MAS DE 200 Y MENOS DE 500 III.-MENOS DE 200 copias

• INHIBIDORES DE TRANSCRIPTASA INVERSA -no análogo de los Nucleosidos- NNRTI• De la ZIDOVUDINA AZT cual es el efecto indeseable? PROBLEMAS MEDULARES, anemia severa, trombocitopenia,

leucopenia

• DE LA EFEVIRENZ cual es el efecto indeseable? • Insomnio y pesadillas (DEPRESIÓN)

• INHIBIDORES DE PROTEASAS • Del Indinavir efecto indeseable mas frecuente: FORMADOR DE CÁLCULOS cólico renoureteral• Ritonavir + lopinavir : Es el caletra efecto sinérgico

Page 61: Preguntas Infectologia

VIH• Profilaxis (exposición en trabajadores de la salud)(exposición SEXUAL) que maniobras o

procedimientos?• Tiene una posibilidad de infección de 1:300 SE REALIZA UN ELISA con fines legales, ( AL MES)… si se

considera EXPOSICIÓN DE BAJO RIESGO dar AZT+3TC X UN MES se REALIZA ELISA SI ES NEGATIVO SE SUSP… si es exposición de alto riesgo AZT+3TC +UN TERCER MEDICAMENTO…. NO DAR TX A LAS 72 POSTEXPOSICION

• Copias de CD4?• Entre 50 y 100 copias en el momento del dx: MVAIC, CMV, Linfomas del SNC• MENOS DE 200: Criptocococis, toxoplasmosis, NEUMOCISTIS JIROVECCI• Entre 200 y 500: LENGUA VELLOSA, Sarcoma de Kaposi, TB Pulmonar, Infección por virus herpes,

candidiosis

• Px con HIV y tiene exposición con px con tb?• Profilaxis: PPD MAS DE 5MM ISONIACIDA + RIFAMPICINA + PIRAZINAMIDA POR 12 MESES +

PIRIDOXINA

• En que consiste el síndrome retroviral agudo?1) Paciente con infección de VIH con SÍNTOMAS A CORTO PLAZO (de 2 A 3 SEMANAS y sucede en el

40%de los casos) Y CONSISTE EN FIEBRE ADENOPATÍA, RASH CUTÁNEO COLOR SALMON Y HEPATOMEGALIA… CUADRO MUY PARECIDO AL DE LA MONONUCLEOSIS INFECCIOSA,

2) DX ELISA A LAS 6 SEMANAS Y 3 MESES si sale positivo se realiza el WESTERN BLOT PARA CONFIRMARLO

Page 62: Preguntas Infectologia

VIH• Combinaciones aprobadas:I. Tenofovir (TDF)+ emtricitabina (FTC) y efavirenz (EFV)II. Atazanavir (ATV) + ritonavir (rtv)+tenofovir (TDF) / emtricitabina (FTC) (si no funciona el primer

esquema)III. Caletra +tenofovir (TDF) / emtricitabina (FTC) (El caletra se puede cambiar con AZT + 3TC)

• Efectos colaterales?I. Zidovudina….. ANEMIA por hipoplasia medular II. Nerfinavir…. DIARREAIII. Didanosina…. PANCREATITIS IV. Zalcitabina….. NEUROPATÍA PERIFÉRICAV. Estaduvidina…. NEUROPATÍA PERIFÉRICA (lamivudina +zidovudina) 3TC….. Neuropatia periféricaVI. Abacavir….. FIEBRE RASH, HIPERSENSIBILIDAD FATAL VII. Tenofovir…. TOXICIDAD RENALVIII. Saquinavir…. LÍPIDOS ELEVADOS

• Px con embarazo + VIH Cual es el tx?

I. ZIDOVUDINA (2do y 3er trimestre)… se continua en el trabajo de parto (para evitar la infección vertical)

II. POSPARTO (NO LACTANCIA 10-20 tiene posibilidad de infectar) 2da opción LAMIVUDINA

Page 63: Preguntas Infectologia

VIH

• Cuales son parte del grupo de fármacos INHIBIDORES DE PROTEASA en VIH?

• R = SaquinAVIR, ritonAVIR, indinAVIR.

• Cuales son parte del grupo de fármacos INHIBIDORES DE LA FUSIÓN?• R = Enfubitida, que entre sus efectos adversos se encuentra la

hipersensibilidad y neumonía bacteriana

• Cuales son parte del grupo de fármacos que inhiben la TRANSCRIPTASA INVERSA en VIH?

• R = ZIDOVUDINA, didanosINA, zalcitabINA.

Page 64: Preguntas Infectologia
Page 65: Preguntas Infectologia
Page 66: Preguntas Infectologia
Page 67: Preguntas Infectologia
Page 68: Preguntas Infectologia
Page 69: Preguntas Infectologia
Page 70: Preguntas Infectologia
Page 71: Preguntas Infectologia

CASO CLINICO

-A 19-year-old man has donated blood for the first time. Despite having no risk factors for human immunodeficiency virus (HIV) infection, his blood tests positive for HIV by enzyme immunoassay (EIA). Which of the following statements is correct?

• (A) EIA is currently the most specific test for HIV• (B) he might have a false-positive secondary to an unsuspected

collagen-vascular disease• (C) he has a 75% chance of truly being infected with HIV• (D) EIA is an excellent screening test• (E) a Western blot test would be more sensitive

Page 72: Preguntas Infectologia

-EIA is an excellent screening test for HIV infection as it is positive in over 99.5% of cases. However, it lacks specificity, and in low risk populations, only about 10% of EIA positive results are true positives. Recent influenza vaccination, acute viral infections, and liver disease are common causes for false positives. The Western blot test is more specific and is the usual confirmatory test, although even more specific tests are now available.

Page 73: Preguntas Infectologia

ASPERGILOSIS• Como se adquiere la aspergilosis?• R = Por inhalación de ESPORAS DEL HONGO, las cuales se encuentran en HOJAS SECAS, GRANEROS

DE MAIZ, ESTIÉRCOL Y VEGETALES EN DESCOMPOSICIÓN.

• En los pacientes con rinitis crónica por aspergilus, que datos de laboratorio encuentras en el moco?

• R = Rico en EOSINOFILOS y cristales de CHARCOT-LEYDEN

• En que consiste la aspergilosis pulmonar endobronquial saprofita o aspergiloma?• R = En el CRECIMIENTO DEL HONGO dentro de las cavidades pulmonares, SECUNDARIAS

GENERALMENTE A TB, SARCOIDOSIS, HISTOPLASMOSIS O BRONQUIECTASIAS.

• Como dx aspergilus?• R = CULTIVO DE SABOURAD, detección de ANTICUERPOS, galactomanano en suero (chbts en su

pared celular) y biopsia

• Cual es el tratamiento de aspergilus?• R = Esteroides, ANFOTERICINA B, LOBECTOMÍA EN CASO DE ASPERGILOMA o debridacion de

tejido infectado.

• Cual es el tratamiento de elección en caso de toxicidad secundaria al DESOXICOLATO de ANFOTERICINA B en caso de aspergilosis invasiva?

• R = VORICONAZOL 6mg/kg/dia.

Page 74: Preguntas Infectologia
Page 75: Preguntas Infectologia

BLASTOMICOSIS• Cual es el agente causal de blastomicosis?• R = BLASTOMYCES DERMATITIDIS

• Cuales son las manifestaciones clínicas de blastomicosis?• R = PULMONAR Y CUTÁNEA. NEUMONÍA aguda que no responde al tratamiento y CURA

ESPONTÁNEAMENTE. Las LESIONES CUTÁNEAS predominan en las ZONAS EXPUESTAS, característicamente son PAPULAS, NÓDULOS O PLACAS BIEN DELIMITADAS, únicas o múltiples, NO DOLOROSAS NI PRURIGINOSAS, que evolucionan a lesiones verrugosas, costras o ulceras.

• Como diagnosticas blastomicosis?• R = De ELECCIÓN CULTIVO pero tarda mucho, se pueden identificar organismos con la

TINCIÓN DE PLATA, METENAMINA O ACIDO PERIODICO DE SCHIFF en la biopsia de los tejidos infectados o en la citología de esputo. Donde se observan CÉLULAS LEVADURIFORMES CON YEMAS DE BASE AMPLIA.

• Cual es el tratamiento de blastomicosis?• R = En la forma LEVE - ITRACONAZOL, en la forma GRAVE - ANFOTERICINA B

Page 76: Preguntas Infectologia

CANDIDOSIS• Como se manifiesta la CANDIDIASIS DIGESTIVA y como se trata?• R = Se ASOCIA a lesiones ORALES Y PERIANALES, produce DIARREA CON MOCO, el

diagnostico se realiza con el EXAMEN DIRECTO DE LAS HECES CON KOH y se trata con NISTATINA ORAL.

• Como se manifiesta la CANDIDIOSIS URINARIA y como se trata?• R = Se presenta como CISTITIS, pielonefritis, BOLAS FÚNGICAS RENALES, ABSCESOS RENALES

o necrosis de las papilas. Se trata con fluconazol o ANFOTERICINA B.

• Como se diagnostica CANDIDIOSIS PULMONAR y como se trata?

• R = Cuando realizas HEMOCULTIVOS POSITIVOS o ANTÍGENO MANAN DEL SUERO. El tratamiento se realiza con ANFOTERICINA B, en los pacientes que no la toleran se utiliza FLUCONAZOL.

• Como se manifiesta la ENDOCARDITIS por candidiosis?• R = FIEBRE persistente, SOPLOS CARDIACOS y ESPLENOMEGALIA, en algunos casos hay

embolización a grandes arterias, como la iliaca o femoral. LA VÁLVULA MAS AFECTADA ES LA MITRAL.

Page 77: Preguntas Infectologia

CANDIDOSIS

• Como diagnosticas ENDOCARDITIS por cándida?• R = Ecocardiagrafia, hemocultivos seriados y detección de

ANTÍGENO MANAN en suero?

• Cual es el tratamiento de endocarditis por cándida?• R = RESECCIÓN DE VÁLVULA afectada y ANFOTERICINA B

• Como se presenta la candidiasis diseminada aguda o sepsis?• R = Se presenta en PACIENTES NEUTROPENICOS, CON FIEBRE,

LEUCOCITOSIS, TROMBOCITOPENIA, inestabilidad hemodinámica y choque.

Page 78: Preguntas Infectologia

CRIPTOCOCOCIS (AVES, PALOMAS)

• Cual es el agente causal de la criptococosis y donde se alberga?• R = CRYPTOCOCCUS NEOFORMANS, se encuentra en HECES DE

PALOMAS (CUIDADORES DE AVES) O NIDOS

• Como diagnosticas criptococosis?

• ANTÍGENO MANAN en suero o biopsia. • En LCR con TINCIÓN DE TINTA CHINA

• Como se trata la criptococosis?• R = ANFOTERICINA B y 5-fluorocitosina

Page 79: Preguntas Infectologia

HISTOPLASMOSIS (CAVERNAS)

• Donde se alberga el Histoplasma capsulatum?• R = En forma de moho en suelo RICO EN NITRÓGENO, como el de los

GALLINEROS, o donde duermen las aves y CAVERNAS DE MURCIÉLAGOS

• Como diagnosticas histoplasmosis?• R = El método de elección es el CULTIVO, pero tarda mucho y haces

frotis de los fluidos o tejidos infectados, las muestras para cultivo en HISTOPLASMOSIS DISEMINADA SE OBTIENEN DE MEDULA ÓSEA, HÍGADO, GANGLIOS LINFÁTICOS O LESIONES MUCOCUTANEAS.

• Cual es el manejo de la hispoplasmosis?• R = Anfotericina B

Page 80: Preguntas Infectologia

MUCORMICOSIS• Cual es el agente etiológico de la mucormicosis?• R = MUCOR, ABSIDIA, RIZOPUS denominado ficomicetos.

• Como se manifiesta la infección de los senos paranasales en mucormicosis?

• R = Predominantemente a PACIENTES CON DM mal tratada, con FIEBRE, congestión nasal, SECRECIÓN NASAL SANGUINOLENTA, DOLOR SINUSAL Y DIPLOPÍA, a la exploración física se observa DISMINUCIÓN DE LOS MOVIMIENTOS OCULARES, quemosis y PROPTOSIS y SI INVADE EL SENO FRONTAL PRODUCE COMA.

• Como se manifiesta la mucormicosis gastrointestinal?• R = Se disemina al peritoneo produciendo la muerte en 70 días

• Como se diagnostica mucormicosis?• R = FROTIS EN FRESCO donde se ven HIFAS HIALINAS o en BIOPSIAS donde se observa

NECROSIS ISQUÉMICA o hemorrágica

• Cual es el manejo de la mucormicosis?• R = El tratamiento consiste en altas dosis de ANFOTERICINA B, ASEO QUIRÚRGICO DE LOS

TEJIDOS.

Page 81: Preguntas Infectologia
Page 82: Preguntas Infectologia

COCCIDIOIDOMICOSIS• Cuales son los signos y síntomas de la coccidioidomicosis?• R = TOS, RINORREA, febricula, dolores musculares y en la RADIOGRAFÍA DE

TÓRAX CON CAVITACIÓN.

• - A 43-year-old man developed a cough shortly after returning from a 1-month hiking trip in California. While there, he was hiking in the central California valleys. During his trip, he had developed a “flu-like” illness consisting of fever, cough, and muscle pains, which resolved spontaneously. A CXR shows a thin-walled cavity in the right upper lobe, and the sputum reveals fungal elements. Which of the following is the most likely causative organism?

• (A) ringworm• (B) Cryptococcus neoformans• (C) Candida albicans• (D) mycobacteria• (E) coccidioidomycosis

Page 83: Preguntas Infectologia

- Coccidioidomycosis is the usual cause of pulmonary cavitation resulting from fungal infection. A rarefaction may be demonstrable in a pneumonic lesion within 10 days of onset

Page 84: Preguntas Infectologia

PALUDISMO• Cual es el agente causal de paludismo o malaria?

• R = PARASITOSIS intracelular eritrocitaria por PLASMODIUM, que se transmite por el

MOSQUITO ANOPHELES

• Cuantas especies de plasmodium existen?• R = 4, P. falciparum, P. vivax, P. ovale y P. malariae

• Cual es el ciclo biológico del plasmodium?• R = Los mosquitos se infectan al ingerir sangre infectada con micro y macrigametocitos.

Dentro del mosquito, se multiplica sexualmente en su intestino, produciendo los ooquistes, y al madurar se rompen liberando ESPOROZOITOS, que migran a las glándulas salivales del mosquito y son la forma infectante en los seres humanos. Ya en el TORRENTE SANGUÍNEO viajan a hígado donde se reproducen y forman los MEROZOITOS que tienen la capacidad de infectar los eritrocitos.

• Que produce la lisis eritrocitaria al contraer paludismo?• R = La PLASMEPSINA, que es una proteasa del acido aspartico que DEGRADA LA

HEMOGLOBINA.

Page 85: Preguntas Infectologia

PALUDISMO• Cual es el tipo de genero en paludismo mas frecuente?• R = P. FALCIPARUM, es la mas frecuente y de peor pronostico con fiebre >

40

• Como se diagnostica el paludismo?• R = OBSERVACIÓN DE PARASITO EN FROTIS SANGUÍNEO O GOTA

GRUESA. La TINCIÓN DE GIEMSA permite observar las granulaciones de Schuffner.

• Como se trata el paludismo?• R = QUININA Y CLOROQUINA. El SULFATO DE QUININA se usa en

RESISTENCIA A CLOROQUINA

Page 86: Preguntas Infectologia
Page 87: Preguntas Infectologia

BORRELIOSIS O ENFERMEDAD DE LYME

• Cual es el agente causal de la enfermedad de Lyme?

• R = Espiroqueta BORRELIA BURGDORFERI GRAM (-) de color violeta con tinción Giemsa que se CULTIVA EN MEDIO NOGUCHI. Se adquiere por MORDEDURA DE GARRAPATA del genero IXODES.

• Cual es el cuadro clínico de la enfermedad de Lyme?• R = Se divide en 3 etapasI. ERITEMA CRÓNICO O MIGRANS, pápulas eritematosas que se EXTIENDEN DE

FORMA CENTRIFUGA DEJANDO LA PARTE CENTRAL DEL CUERPO SANA, acompañada de síntomas generales como astenia, adinamia, fiebre, mialgias y cefalea.

II. Alteraciones neurológicas, cardiacas y reumáticasIII. POLIARTRITIS MIGRATORIA, alteraciones dermatológicas y neurológicas

• Como se diagnostica la enfermedad de Lyme?• R = ELISA o PCR

Page 88: Preguntas Infectologia

BORRELIOSIS O ENFERMEDAD DE LYME

• Tratamiento de elección para enf de Lyme?• R = DOXICICLINA por mes y medio

• Cual es el tratamiento para borreliosis en su etapa precoz?• R = DOXICICLINA.

• Cual es el tratamiento para borreliosis en su etapa tardía?• R = CEFTRIAXONA

Page 89: Preguntas Infectologia

LEPTOSPIROSIS• Cual es el vector de la leptospirosis, que enzimas produce y que nutrientes necesita?• R = Leptospira interrogans en ORINA DE RATA. PRODUCE CATALASA Y HIALURODINASA y requieren TIAMINA

B1 Y VITAMINA B12 PARA SU DESARROLLO

• Cuales son las manifestaciones clínicas de la Leptospirosis?• R = De inicio súbito, con FIEBRE, derrame conjuntival, epistaxis, MIALGIAS, cefalea intensa, EXANTEMA Y

HEPATOMEGALIA.

• Que es el síndrome de Weil en Leptospirosis?• R = Al cursar la primera fase, viene una ASINTOMÁTICA para posteriormente PRESENTARSE LEPTOSPIROSIS

ANICTERICA.

• Como se manifiesta el síndrome de Weil?• R = CON FIEBRE, CEFALEA, MIALGIAS, EXANTEMA, miocarditis, ICTERICIA, insuficiencia hepática,

INSUFICIENCIA RENAL, FENÓMENOS HEMORRÁGICOS, LEPTOSPIURIA y daño al SNC (MENINGITIS ASEPTICA).

• Cual es la triada clásica del síndrome de Weil?• R = Daño en HÍGADO, RIÑÓN y SNC.

• Cual es el tratamiento de la Leptospirosis?• R = PENICILINA, estreptomicina o tetraciclinas

Page 90: Preguntas Infectologia

TULAREMIA• Cual es el agente causal de Tularemia?

• R = FRANCISELLA TULLARENSIS POR PICADURA DE GARRAPATA

• Cual es el cuadro clínico de Tularemia?• R = De inicio súbito con FIEBRE, ASTENIA, ADINAMIA, hiporexia, ataque al estado

general, posteriormente hay diversas formas clínicas: ULCEROGANGLIONAR, manifestada con adenomegalias y ulceras cutáneas; TIFOIDE GLANDULAR, donde se afecta el pulmón, ganglios, hígado y bazo. SE PRESENTA CON NEUMONÍA, NECROSIS VISCERAL Y SEPSIS.

• Como diagnosticas Tularemia?

• R = En medio de CULTIVO DE AGAR SANGRE cisteína o ELISA.

• Como tratas la Tularemia?

• R = ESTREPTOMICINA, tetraciclina.

Page 91: Preguntas Infectologia
Page 92: Preguntas Infectologia

RICKETSIA

Page 93: Preguntas Infectologia
Page 94: Preguntas Infectologia

FIEBRE Q

Page 95: Preguntas Infectologia
Page 96: Preguntas Infectologia

LEISHMANIASIS• Que vectores transmiten leishmaniasis?• R = MOSQUITOS del genero LUTZOMYA Y PHLEBOTOMUS, LEISHMANIA pertenece a la familia del

tripanosoma

• Cual es el ciclo biológico de la leishmaniasis?• R = Al picar el mosquito a un mamífero infectado, ingiere los macrófagos parasitados, que posteriormente

se destruyen, dejando libre al parasito en su forma flagelar e infectante (promastigote), se reproducen por fision binaria longitudinal, y migran hacia la porción anterior del aparato digestivo, hasta la probocide (boca). La infección se produce al alimentarse de un nuevo huésped, inocula los PROMASTIGOTES, que al ser fagocitados se transforman en AMASTIGOTES, donde se multiplican, causan la lisis de las células que los contienen e infectan a otras células.

• En la leishmaniasis viceral o Kala-Azar a que células afecta el parasito?• R = A los MACRÓFAGOS del sistema retículo endotelial

• Como se manifiesta la leishmaniosis cutánea?• R = En el sitio de inoculación, PAPULA ERITEMATOSA/PRURIGINOSA – NODULO – ULCERA PIEL/TEJ. CEL.

SUB – EXUDA LIQ. SEROHEMATICO – ADENOPATÍA REGIONAL.

• Como se manifiesta la leishmaniosis mucocutanea o espundia?• R = ULCERA CUTÁNEA – pasan years – LESIÓN OBSTRUCTIVA ÚNICA/MULTIPLE – LARINGE/TABIQUE

NASAL/ANO Y VULVA.

Page 97: Preguntas Infectologia

LEISHMANIASIS• Como se manifiesta la leishmaniasis viceral o Kala-Azar (fiebre negra)?• R = Hay HIPERPIGMENTACIÓN CUTÁNEA DE LAS EXTREMIDADES, mal estar general, DOLOR ABDOMINAL, HEPATO-

ESPLENOMEGALIA, LINFADENOPATIAS, DIARREA es fatal si no se trata adecuadamente.

• Como diagnosticas leishmaniasis?• R = VER EL AMASTIGOTE EN BIOPSIAS DE PIEL, lesiones mucosas, hígado, ganglios linfáticos, medula ósea tenidas con

Giemsa. En los CULTIVOS de los tejidos se busca el PROMASTIGOTE.

• Con que prueba de laboratorio diagnosticas leishmaniasis?• R = INTRADERMORREACCIÓN DE MONTENEGRO y ELISA.

• Cual es el tratamiento de la leishmaniasis viceral?

• R = Para la FORMA VISCERAL el tratamiento DE ELECCIÓN ES ESTIBOGLUCONATO DE SODIO O ANTIMONIATO DE GLUTAMINA IM POR 50 DÍAS.

• Que medicamentos empleas en caso de que la leishmaniasis visceral no responda a los antiamoniales?• R = ANFOTERICINA LIPOSOMAL POR 5 DÍAS Y EL ISETHIONATO DE PENTAMIDINA.

• Que medicamento utilizas en leishmaniasis cutánea?

• R = ISETIONATO DE PENTAMIDINA, paromomicina APLICACIÓN TÓPICA 2 VECES AL DÍA POR 15 DÍAS.

Page 98: Preguntas Infectologia
Page 99: Preguntas Infectologia

GIARDIASIS• Cual es el agente causal de giardiasis y a que gpo de edad afecta comúnmente?• R = GIARDIA INTESTINALIS de la clase MASTIGOPHORA, de la FAMILIA HEXANITIDAE y afecta a

menores de 5 años

• Cual es el ciclo biológica de la giardiasis?• R = Los TROFOZOITOS presentes en el duodeno, avanzan por el intestino y debido a la escasa

cantidad de agua en el colon, cambian de forma a su estado quístico. El quiste permanece como la forma infectante y es ELIMINADO CON LA MATERIA FECAL. Al ser INGERIDO POR EL HUÉSPED, pasa por el tubo digestivo y los ácidos gástricos rompen la pared liberando al trofozoito, que nuevamente pasa al duodeno, donde madura, se fija a la pared intestinal y se replica por fision binaria longitudinal y es arrastrado con la materia fecal.

• Que déficit nutricional puede ocasionar la giardiasis?• R = Déficit de vitaminas KADE, B12 Y ACIDO FÓLICO.

• Como se manifiesta la giardiasis?• R = En niños cursa asintomática. NAUSEA , VOMITO, DIARREA CON MOCO Y ESTEATORREA.

Page 100: Preguntas Infectologia

GIARDIASIS

• Como diagnosticas giardiasis?• R = Observación de QUISTE EN HECES, ELISA.

• Que componente de los trofozoitos de giardiasis te causa hipotrofia de las vellosidades intestinales

• R = El DISCO SUCTOR

• Cual es el tratamiento de elección para la giardiasis?• R = El TINIDAZOL con dosis única de 2 gr. METRONIDAZOL. La

furazolidona causa hemolisis en pacientes con déficit de 6-GDP.

Page 101: Preguntas Infectologia

AMEBOSIS• Cual es el agente causal de la Amebosis?• R = Entamoeba dispar y ENTAMOEBA HYSTOLITICA.

• Como se desplaza el quiste de la E. Hystolitica?• R = Pseudopodos

• Cual es el ciclo biológico de la entamoeba hystolitica?• R = Ingestión de quistes maduros, desenquistamiento en intestino delgado y los

TROFOZOITOS liberados tetranucleados multiplican sus núcleos para formar una ameba de ocho núcleos, que posteriormente se fragmenta en ocho amebas, llamadas amébulas meta quísticas, que maduran a trofozoitos y migran a colon, donde se alimentan de bacterias y restos celulares; se reproducen por fisión binaria y producen quistes que se eliminan por las heces.

• Como se unen los parásitos del intestino grueso de E. Hystolitica?• R = Mediante la lectina

Page 102: Preguntas Infectologia

AMEBOSIS• Cual es el cuadro clínico abdominal de la amebiasis por E. Hystolitica?1. COLITIS AMEBIANA AGUDA manifestada por dolor abd tipo cólico, tenesmo y EVACUACIONES

DIARREICAS CON MOCO Y SANGRE. 2. COLITIS FULMINANTE con mortalidad mayor al 50% se presenta con diarrea sanguinolenta intensa,

ataque al estado general, fiebre, dolor abdominal intenso y PERFORACIÓN INTESTINAL con choque séptico secundarios a la extensión de la ulcera hasta la serosa.

3. AMEBOMA O GRANULOMA AMEBIANO que cursa con DIARREA SANGUINOLENTA Y OBSTRUCCIÓN INTESTINAL.

• Cual es el cuadro clínico de la amebosis cutánea?1. ULCERAS FUGEDENICAS (de COLOR PURPURA, de crecimiento rápido, MUY DOLOROSAS y

ABUNDANTE MATERIAL NECRÓTICO), 2. LESIONES VEGETANTES (friables, de evolución subaguda, LOCALIZADAS EN LOS PLIEGUES

CUTÁNEOS), 3. ENTAMOEBOSIS (se presenta por una reacción de hipersensibilidad, CON LESIONES SEMEJANTES A

URTICARIA.

• Como se manifiesta el absceso hepático amebiano?• R = De inicio agudo, CON HEPATOMEGALIA Y DOLOR EN HIPOCONDRIO DERECHO QUE SE IRRADIA

HACIA EL HOMBRO.

Page 103: Preguntas Infectologia

AMEBOSIS• Como diagnosticas amibiasis?• R = COPROPARISOTOSCOPICO EN FRESCO, hematoxilina férrica o TINCIÓN

DE GOMORI, en busca de TROFOZOITOS, ELISA. En la piel se busca trofozoito.

• Cual es el tratamiento de amibiasis?1) METRONIDAZOL, TINIDAZOL. 2) Dehidrohemetina y emetina son amebicidas de la pared intestinal, pero

no sobre la luz intestinal siendo estos YODOQUINOL y paramomicina.

• Que medicamento es eficaz para la amebiasis hepática?• R = CLOROQUINA

Page 104: Preguntas Infectologia
Page 105: Preguntas Infectologia

TRIPANOSOMIASIS AFRICANA O ENFERMEDAD DEL SUENO

• Cual es el agente etiológico de la TRIPANOSOMIASIS AFRICANA O ENFERMEDAD DEL SUEÑO y cual es su vector?

• R = Tripanosoma BRUCEI, Tripanosoma RHODESIENSE y Tripanosoma GAMBIENSE. Su

vector es la MOSCA TSE TSE.

• Cual es el agente causal de la tripanosomiasis americana o ENFERMEDAD DE CHAGAS y cual es su vector?

• R = TRIPANOZOMA CRUZI y su vector es la chinche

• Cual es el ciclo biológico de los tripanosomas?• R = Al alimentarse de un individuo con la infección, los vectores ingieren el amastigote,

que se multiplica dentro del tracto digestivo y las FORMAS INFECCIOSAS (TRIPOMASTIGOTE) son eliminadas en las heces; la infección se produce al depositar las heces en una herida de la piel antes de picar en las membranas mucosas o en la conjuntiva. Posteriormente, se multiplican en el sitio de la entrada, y entran a la sangre como tripomastigotes circulantes e invaden a las células, CON PREDILECCIÓN POR EL MIOCARDIO, MUSCULO LISO Y SNC.

Page 106: Preguntas Infectologia

TRIPANOSOMIASIS O ENFERMEDAD DE CHAGAS

• Como se manifiesta la tripanosomiasis o ENFERMEDAD DE CHAGAS POR T. CRUZY?• R = En la piel, en el sitio de entrada se observa CHAGOMA que es PARECIDO A LA FURUNCULOSIS CON

LINFADENOPATIA REGIONAL. En conjuntiva, edema bipalpebral unilateral, CONJUNTIVITIS Y LINFADENITIS PRE AURICULAR (SIGNO DE ROMANA) CON FIEBRE Y ESPLENOMEGALIA LEVE. CRONICAMENTE se manifiesta por INSUFICIENCIA CARDIACA PREDOMINANTE DEL LADO DERECHO, con arritmias ventriculares y episodios de tromboembolia sistémica o pulmonar. MEGA ESÓFAGO Y MEGACOLON con síntomas de disfagia, constipación intensa, hipertrofia parotidea.

• Como se diagnostica la tripanosomiasis americana o ENFERMEDAD DE CHAGAS?• R = En la FASE AGUDA se observa con la TINCIÓN DE GIEMSA de sangre anti coagulada. Diagnostico

serológico se basa en hemaglutinación indirecta o ELISA.

• Como se diagnostica la TRIPANOSOMIASIS AFRICANA?• R = Para T. GAMBIENSE por medio de PRUEBAS DE AGLUTINACIÓN y para T. RHODESIENSE es mediante la

observación del parasito en la tinción de GIEMSA.

• Cual es el tratamiento de elección para la TRIPANOSOMIASIS AMERICANA O CHAGAS?

• R = NIFURTIMOX por 30-90 DÍAS o BENZIDIMAZOL de 30-90 DIAS.

• Cual es el tratamiento de la TRIPANOSOMIASIS AFRICANA?• R = Para T. GAMBIENSE ES PENTAMIDINA POR 10 DÍAS. Para T. RHODESIENSE ES LA SURAMINA POR 7

DÍAS.

Page 107: Preguntas Infectologia

BABEIOSIS• Cual es el agente causal de la babeiosis y cual es su vector?• R = Enfermedad INTRAERITROCITARIA, causada por PROTOZOARIO BABEZIA MICROTI Y DIVERGENS, transmitida

por el vector que es una GARRAPATA de nombre IXODES SCAPULARIS

• Cual es el ciclo biológico de la babesiosis?• R = Cuando las GARRAPATAS I. SCAPULARIS se alimentan de un animal infectado, ingieren los parásitos en el intestino,

se produce la esquizogonia y posteriormente los merozoitos que infectan las células epiteliales del vector, y son transmitidas a glándulas salivales por la vía linfática y alimentarse del nuevo huésped le transmiten la infección.

• Como se observan los parásitos de la babeiosis en los eritrocitos?• R = INCLUSIONES EN FORMA DE CRUZ lo que causa ANEMIA HEMOLÍTICA

• Cual es el cuadro clínico de la babeiosis?• R = Clínicamente se manifiesta por FIEBRE, ANEMIA HEMOLÍTICA, cefalea, diaforesis, MIALGIAS, artralgias, fatiga,

ictericia, ESPLENOMEGALIA.

• Como diagnosticas la babeiosis?• R = Identificación del PARASITO INTRAERITROCITARIO EN FROTIS CON TINCIÓN DE GIEMSA. ANTICUERPOS ENTRE 1ERA

Y 4TA SEMANA. En bh se observa ANEMIA, TROMBOCITOPENIA y LEUCOCITOSIS leve.

• Cual es el tratamiento de la babeiosis?

• R = Puede resolverse sin tratamiento. Para LA BABEIOSIS SINTOMÁTICA ES CON QUINIDINA MAS CLINDAMICINA.

• Que tipo de babeiosis tiene peor pronostico?• R = B. DIVERGENS con 40% de muertes.

Page 108: Preguntas Infectologia

TENIASIS• Cual es el agente causal de la teniasis?• R = T. saginata y T. solium

• Cuales son las partes estructurales que componen a la tenia?• R = En su extremo anterior tienen una estructura llamada ESCÓLEX (CABEZA), seguida de un cuello. En su

escólex presentan cuatro ventosas orales. En sus ventosas tiene un róstelo, con una corona doble de ganchos (escolices). El CUERPO (ESTRÓBILO) se conforma por varios segmentos llamados PROGLOTIDOS, que en su interior tienen ÓRGANOS SEXUALES.

• Cuanto miden los adultos de T. SAGINATA?• R = 2-7 METROS de largo

• Cuanto puede llegar a medir la T. SOLIUM?

• R = 15-18 METROS

• Cual es el ciclo biológico de la teniasis?• R = La teniasis intestinal es causada por la INGESTA DE CISTICERCOS. Al legar a intestino delgado, se unen

a la pared intestinal. De los 2 a los 4 meses de la infección, alcanzan su madurez, caracterizada por la presencia de proglotidos gravidos que liberan huevos y se excretan por las heces. La cisticercosis ocurre por la ingestión de los alimentos o agua contaminados con excremento humano que contiene huevos o proglotidos. Los huevos se abren en el intestino, liberando a las oncoesferas, que invaden la pared intestinal pasando al torrente sanguíneo y linfático para migrar hacia musculo estriado, cerebro e hígado.

Page 109: Preguntas Infectologia

TENIASIS• Cual es el cuadro clínico de la teniasis?• R = Normalmente es asintomática. Cuando cursa con síntomas son: PLENITUD

ABDOMINAL, FATIGA, DOLOR LEVE OCASIONAL E HIPOREXIA.

• Que déficit vitamínico cursa con teniasis?• R = Déficit de VITAMINA B12

• Cual es el cuadro clínico de CISTICERCOSIS en el SNC?• R = CRISIS CONVULSIVAS, cefaleas, vomito, HIPERTENSIÓN INTRACRANEAL.

• Como diagnosticas la teniasis?• R = COPROPARASITOSCOPICO SE OBSERVAN HUEVOS, ELISA.

• Cual es el tratamiento para teniasis?• R = PRAZICUANTEL a DOSIS ÚNICA DE 10 MG/KG. Ninguna droga mata a los huevos

de tenia solium. NICLOSAMIDA para TENIA SAGINATA

Page 110: Preguntas Infectologia

ASCARIASIS• Cuanto mide el macho y la hembra de áscaris?• R = La HEMBRA es mas grande mide 25-30 CM y el MACHO 25 CM.

• Cual es el ciclo biológico de la ascariasis?• R = La infección ocurre por la ingestión de huevos maduros. Los huevos al ser

eliminados por las heces, no tienen envoltura y son infecciosos a las 2-3 semanas en el suelo o el agua. Al ser ingeridos eclosionan en el intestino, las larvas inmaduras penetran la pared intestinal y SE DIRIGEN HACIA LOS ALVEOLOS A TRAVÉS DEL TORRENTE SANGUÍNEO, posteriormente, ASCIENDEN POR LOS BRONQUIS HASTA LA FARINGE, DONDE SON DEGLUTIDOS COMPLETANDO SU DESARROLLO EN EL INTESTINO DELGADO.

• Cual es el cuadro clínico de la ascariasis?• Pulmonar: SÍNDROME DE LOFFLER (NEUMONÍA EOSINOFILICA) POR MIGRACIÓN

ALVEOLAR, CON TOS PRODUCTIVA, DISNEA, sibilancias, dolor retroesternal, FIEBRE. • Intestino, cursa asintomático pero puede ocasionar APENDICITIS, OBSTRUCCIÓN DE

CONDUCTOS BILIARES.

Page 111: Preguntas Infectologia

ASCARIASIS• Como diagnosticas ascariasis?• R = Hallazgo de HUEVOS POR COPROPARASITOSCOPIO POR CONCENTRACIÓN DE

FLOTACIÓN (FAUST) O POR SEDIMENTACIÓN (RITCHIE) QUE ES CUALITATIVO. Método cuantitativo de Stoll y Kato-Katz/ Kato-Miura para determinar numero de parásitos.

• Cual es el tratamiento de la ascariasis?• R = ALBENZAZOL Y EL PAMOTATO DE PIRANTEL son los medicamentos de elección.

• En que momento del embarazo puedes tratar ascariasis?• R = En el 3ER TRIMESTRE

• Como actúa el pamotato de pirantel?• R = Inhibe la acetilcolinesterasa y bloquea la transmisión de la placa neuromuscular.

• Cuales son algunas de las complicaciones extraintestinales de la ascariasis?• R = Pancreatitis, colangitis, colecistitis, absceso y perforación hepática, obstrucción de

vías respiratorias y rotura del conducto onfalomesenterico.

Page 112: Preguntas Infectologia

OXIUROSIS• Cual es el agente causal de la Oxiurosis?• R = Nematodo ENTEROBIUS VERMICULARIS, tiene la forma de alfiler.

• Cual es el ciclo biológico de la Oxiurosis?• R = Los parásitos adultos SE ENCUENTRAN EN EL CIEGO, donde copulan y una vez que han

terminado LA HEMBRA MIGRA HACIA EL ANO, depositando DURANTE LAS NOCHES SUS HUEVOS EN LA REGIÓN PERIANAL, secretando una sustancia que PRODUCE PRURITO INTENSO, lo que ocasiona el rascado y la contaminación de las manos.

• Cual es el cuadro clínico de la oxiurosis?• R = PRURITO PERIANAL O VULVAR NOCTURNO.

• Como diagnosticas oxiurosis?• R = CON CINTA ADHESIVA PERIANAL observando los parásitos CON TINCIÓN GRAM

• Cual es el tratamiento de la oxiurosis?• R = Se debe tratar a toda la familia con ALBENDAZOL 400 MG/DOSIS ÚNICA

Page 113: Preguntas Infectologia

ESTRONGILOIDOSIS• Cual es el agente causal de la estrongiloidosis?• R = Helminto STRONGYLOIDES STERCORALIS

• Cual es el ciclo biológico de la estrongiloidosis?• R = Los adultos copulan en el yeyuno del huésped, y las hembras fecundadas entran a la pared intestinal y permanecen en la

submucosa, los huevos larvados se desprenden de la submucosa y en la luz intestinal liberan a la larva rabditoide, pudiendo transformarse en larvas filariformes las cuales pueden penetrar el intestino pasando a la circulación donde digieren a diferentes tejidos.

• De donde se obtiene la estrongiloidosis?• R = De CAMINAR DESCALZO, atraviesan la piel causando infección

• Cual es el cuadro clínico de la estrongiloidosis?1) En el sitio DONDE PENETRA LA LARVA filariforme, se PRESENTAN LESIONES PAPULOSAS, ERITEMATOSAS Y PRURIGINOSAS

CON EDEMA LOCAL E INFLAMACIÓN. 2) Si es en LA REGION PERIANAL SE PRESENTA COMO DERMATITIS PERIANAL RADIADA. 3) Intestinales: se manifiestan por ulceras, DIARREA, DOLOR EPIGÁSTRICO URENTE, meteorismo, melena. 4) Pulmonar: con SÍNDROME DE LOFFLER, con tos, irritación traqueal, fiebre y hemoptisis.

• Como diagnosticas la estrongiloisosis?• R = El COPROPARASITOSCOPICO por concentración- flotación (Faust) se OBSERVAN LAS LARVAS.

• Cual es el tratamiento de la estrongiloidosis?• R = IVERMECTINA o ALBENDAZOL

Page 114: Preguntas Infectologia

TRIQUINOSIS• Cual es el agente causal de la triquinosis?• R = TRICHINELLA SPIRALIS la cual se obtiene al consumir CARNE DE CERDO

• Cual es el ciclo biológico de la triquinosis?• R = Consumiendo carne de cerdo, los jugos gástricos liberan las larvas enquistadas, maduran en el intestino delgado

donde se reproducen. Posteriormente atraviesan la mucosa intestinal y se diseminan por la vía linfática/sanguínea hacia otros tejidos, especialmente musculo estriado pobre en glucógeno y de mayor actividad (diafragma, intercostales, bíceps, cuádriceps, lengua y maseteros) provocando una reacción inflamatoria.

• Cual es el cuadro clínico de la triquinosis?• R = Se manifiesta en 3 fases. 1) Fase intestinal: con dolor mesogastrico tipo cólico, mal estar general, nausea, vomito, cefalea y diaforesis. 2) Fase de migración: edema, fiebre, diaforesis, fotofobia, conjuntivitis,3) Fase muscular: DOLOR MUSCULAR QUE EXACERBA CON EL EJERCICIO. 4) A la ex fis se observan hemorragias en forma de astilla en unas retina y sub conjuntivales.

• Como se realiza el diagnostico la triquinosis?• R = Coproparasitoscopio, biopsia de músculos dolorosos tenidos con hematoxilina y eosina con elevación de la CPK.

• Cual es el tratamiento de la triquinosis?

1) Fase intestinal se da ALBENDAZOL 400 mg/12 x 15 días. 2) Fase muscular es intrahospitalario con antiparasitarios mas AINES.

Page 115: Preguntas Infectologia

FASCIOLOSIS• Cual es el agente causal de la fasciolosis?• R = FASCIOLA HEPÁTICA

• Cual es el huésped intermediario de la fasciola hepática?

• R = Los CARACOLES

• Cual es el ciclo biológico de la fasciola hepática?• R = Al ingerir las metacercarias, llegan al intestino delgado eclosionando, dejan salir las alrcas que atraviesan la pared

intestinal y se dirigen a hígado, hasta llegar a los conductos biliares, donde se establecen para desarrollarse. Al fecundar en el hígado los huevos pasan al duodeno y se expulsan con la materia fecal, posteriormente el opérculo se abre para dejar salir al miracidio, el cual es una larva ciliada, que le permite desplazarse y penetrar a los caracoles pulmonados donde se desarrolla el esporoquiste.

• Cual es el cuadro clínico de la fasciolosis?• R = DOLOR INTESTINAL, FIEBRE ALTA, HEPATO-ESPLENOMEGALIA, ICTERICIA con presencia de microabscesos y necrosis.

• Como diagnosticas la fasciolosis?

• R = EOSINOFILIA ELEVADA, COPROPARASITOSCOPIO SERIADO

• Cual es el tratamiento de elección para la Fasciolosis?

• R = El BIOTINOL, se puede usar DEHIDROHEMETINA, METRONIDAZOL Y PRAZICUANTEL.

Page 116: Preguntas Infectologia

ELEFANTIASIS O FILARIOSIS LINFÁTICA

• Cuales son los agentes etiológicos de la elefantiasis o filariosis linfática?• R = WUCHERERIA BANCROFTI y B. malayi

• Cual es el agente causal de la Filariosis?• R = Microfilaria

• Cual es el vector de la filariosis?• R = MOSQUITOS O MOSCAS.

• Cual es el ciclo biológico de la filariasis?• R = Las microfilarias son ingeridas por insectos hematófagos y transmitidas a los huéspedes

donde se ALOJAN EN SANGRE Y TEJIDO LINFÁTICO

• Cual es el cuadro clínico de la filariasis?• R = FILARIASIS LINFÁTICA: FIEBRE CON/SIN INFLAMACIÓN DE GANGLIOS Y VASOS

LINFÁTICOS, PUEDE HABER EPIDIDIMITIS U ORQUITIS. En la infección crónica causa OBSTRUCCIÓN LINFATICA CAUSANDO EDEMA, HIDROCELE, LINFEDEMA ESCROTAL, VARICES LINFÁTICAS Y ELEFANTIASIS.

Page 117: Preguntas Infectologia

ELEFANTIASIS O FILARIOSIS LINFÁTICA

• Que te provoca la QUILURIA de la filariasis?• R = La ROTURA DE LOS VASOS LINFÁTICOS EN EL TRACTO URINARIO

• Como se manifiesta la oncocercosis transmitida por la inoculación de larvas en la picadura de mosca en filariasis?

• R = El parasito se aloja en la piel formando nódulos con edema y eritema local.

• Cuales son los agentes causales en la oncocercosis o ceguera de los ríos en filariasis?• R = ONCHOCERCA VOLVULUS

• Como diagnosticas la filariasis?• R = Se observan las MICROFILARIAS EN SANGRE Y LINFA. Se toma biopsia de piel.

• Como tratas la filariosis?• R = DIETILCARBAMACEPINA

Page 118: Preguntas Infectologia

ESQUISTOSOMIASIS• Cual es el ciclo biológico de la esquistosomiasis?• R = Los huevos son eliminados por las heces, al tener contacto con el agua se liberan las larvas sin la espina

(miracidios), que nadan hasta encontrar un caracolinvadiendo los músculos donde se transforman en esporocitos, estas salen del caracol en forma de cercarías mismas que penetran en la piel del hombre cuando nada y se alimentan de eritrocitos.

• Cual es el cuadro clínico de la esquistosomiasis?1) DERMATITIS CERCARÍA: DONDE HAY PAPULAS ERITEMATOSAS Y PRURIGINOSAS EN EL SITIO DE LA

INOCULACIÓN. En algunos casos se presentan manifestaciones alérgicas caracterizadas por fiebre, urticaria y eosinofilia.

2) FIEBRE DE KATAYAMA: FIEBRE, MIALGIAS, URTICARIA, tos no productiva, dolor abd, diarrea, eosinofilia y HEPATOESPLENOMEGALIA.

3) CRONICIDAD POR SCHISOTOMA MANSORI O JAPONICUM: DIARREA SANGUINOLENTA y hepatoesplenomegalia.

4) CRONICIDAD POR SCHISOTOMA HEMATOBIUM HAY DISURIA, HEMATURIA Y PROTEINURIA, POSTERIOREMENTE HAY PÓLIPOS EN VEJIGA, GLOMERULONEFRITIS, CISTITIS Y URETRITIS.

• Como diagnosticas esquistosomiasis?• R = COPROPARASITOSCOPICO identificando los huevos, o en la ORINA, se puede realizar biopsia o tinción de

ELISA.

• Cual es el tratamiento de elección de la esquistosomiasis?• R = PRAZICUANTEL 20 MG/KG C/12 HRS

Page 119: Preguntas Infectologia

TRICOCEFALOSIS/ TRICHURIS TRICHIRIA

• Cual es el agente causal de la tricocefalosis y a quienes afecta principalmente?

• R = TRICHURIS TRICHURIA y afecta principalmente a niños.

• Cual es el ciclo biológico de la tricocefalosis?• R = Ingestión de alimentos con huevos larvados. Los huevos eclosionan en el intestino delgado y migran

hacia el ciego, se introduce en la mucosa y deposita sus huevos en el lumen siendo excretados con las heces.

• Cual es el cuadro clínico de la trichuris trichiria?

• R= DOLOR ABDOMINAL TIPO CÓLICO, pujo, tenesmo, diarrea, distensión abdominal y PROLAPSO RECTAL.

• Cual es el cuadro clínico de trichuris en niños?• R = Síndrome disentérico, ANEMIA FERROPENICA, palidez, PROLAPSO RECTAL Y RETARDO EN EL

CRECIMIENTO.

• Como se diagnostica la trichuriasis o tricocefalosis?• R = COPROPARASITOSCOPIO por concentración-flotacion (Faust). También se pueden utilizar los CRISTALES

DE CHARCOT LEIDEN que son los productos de degradación de los eosinofilos.

• Cual es el tratamiento de elección de tricocefalosis?• R = ALBENDAZOL Y HIERRO

Page 120: Preguntas Infectologia

TOXOCARIASIS• Cual es el agente causal de la toxocara?• R = TOXOCARA CANIS, TOXOCARA CATI.

• Cuanto mide el parasito de toxacara?• R = Menor al de Ascaris lumbricoides 5-10 CM

• Cual es el ciclo biológico de la toxocarosis?• R = Huevos eliminados en la materia fecal, son ingeridos por el hombre, intestino delgado, penetran a pared intestinal y

migran por torrente sanguíneo a hígado, pulmones, SNC, corazón y los ojos provocando la FORMACIÓN DE GRANULOMAS EOSINOFILOS.

• Cual es el cuadro clínico de la toxocariasis?• R = El órgano MAS AFECTADO ES EL HÍGADO, donde se producen GRANULOMAS y los PULMONES MANIFESTADA COMO

SÍNDROME DE LOFFLER, cursando con tos, fatiga, disnea, prurito, dolor abdominal, hepatomegalia, esosinofilia, hiperglobulinemia (IgE). La toxocariasis ocular donde hay estrabismo y deterioro visual unilateral donde se observa un granuloma eosinofilico.

• Como realizas el diagnostico de toxocariosis?• R = ELISA. Hipergamaglobulinemia IgE. En LESIONES HEPÁTICAS se observan lesiones HIPOECOICAS.

• Cual es el tratamiento de elección la toxocariasis?

• R = ALBENDAZOL 400 MG CA/12 X 21 DIAS.

Page 121: Preguntas Infectologia

CASO CLINICO• Cual es el cuadro clínico en RN o lactantes de toxoplasmosis cerebral y que

resultados en TAC se observan?• R = The infection has a predilection for the CNS and the eye, and produces

encephalitis in utero. Symptoms can be evident in the first few days of life. Infants born with active disease may have fever, rash, seizures, and hepatosplenomegaly at birth. En la TAC se observan calcificaciones cerebrales

• - A 6-month-old child presents with recurrent seizures and poor development. The evaluation reveals a baby with hydrocephalus, impaired movement of the extremities, hypotonia, and retinal abnormalities. A computerized tomography (CT) scan demonstrates large ventricles and calcified lesions. Which of the following is the most likely diagnosis?

• (A) Tay-Sachs disease• (B) congenital hydrocephalus• (C) kernicterus• (D) toxoplasmosis• (E) congenital neurosyphilis

Page 122: Preguntas Infectologia

HERPES VIRUS SIMPLE 2Cual es la manifestación clínica de herpes virus simple 2?R = VESÍCULAS CON BASE ERITEMATOSA con hipertrofia ganglionar local

-A 23-year-old woman develops vesicular lesions on an erythematous base on her vulvar area. She has tender lymphadenopathy and dysuria as well. Which of the following is the most likely causative organism?

• (A) cytomegalovirus (CMV)• (B) gonococcus• (C) herpes simplex virus type 2 (HSV-2)• (D) Treponema pallidum• (E) varicella zoster

Page 123: Preguntas Infectologia

-HSV-2 genital infections may be associated with fever, malaise, and anorexia. Vesicular lesions usually ulcerate rapidly and become covered with exudate. There is a 90% chance of recurrent symptoms in the first year following a primary infection. HSV-1 genital infections are similar, but the chance of recurrence is less.

Page 124: Preguntas Infectologia

OOFORITIS• Cual es la complicación mas común de parotiditis en las mujeres?• R = Ooforitis

-An 18-year-old woman presents with headache, anorexia, chilly sensations, and discomfort on both sides of her jaw. She has also noticed discomfort in both lower abdominal quadrants. Physical examination reveals bilateral enlarged parotid glands that are doughy, elastic, and slightly tender; with a reddened orifice of Stensen’s duct. Her abdomen is soft with bilateral lower quadrant abdominal tenderness; a temperature of 38.5°C; and a pulse rate of 92/min. Laboratory data show hemoglobin 13 g/dL; hematocrit 40%; white blood cells (WBC) 9000/mL, with 35% segmented neutrophils, 7% monocytes, and 58% lymphocytes. Which of the following is the most likely cause for her abdominal pain and tenderness?

• (A) mesenteric lymphadenitis• (B) oophoritis• (C) gonorrhea• (D) peritoneal metastases• (E) intestinal hyperperistalsis

Page 125: Preguntas Infectologia

-Pain referring to either or both lower quadrants is common when oophoritis is present. Fever usually accompanies oophoritis. Sterility is not a consequence of mumps oophoritis.

Page 126: Preguntas Infectologia

CASO CLINICO- A 23-year-old woman visits your office because of headache, malaise,

anorexia, pain in both sides of her jaw, and discomfort in both lower abdominal quadrants. Physical examination reveals enlarged parotid glands; bilateral lower quadrant abdominal tenderness; a temperatura of 38.7°C; and a pulse rate of 92/min. Serologic testing (IgM) confirms the diagnosis of mumps. Which of the following is the most appropriate treatment for this condition?

• (A) symptomatic• (B) immunization• (C) broad-spectrum antibiotics• (D) sulfonamides• (E) steroids

Page 127: Preguntas Infectologia

- Antibiotics, sulfas, steroids, and mumps convalescent sera are of no value. Mouth care, analgesics, and a bland diet are usually recommended. Glucocorticoids are usually prescribed for orchitis, although definite evidence of their effectiveness is lacking. Prevention via vaccination is the preferred strategy for mumps.

Page 128: Preguntas Infectologia

ETS

Page 129: Preguntas Infectologia
Page 130: Preguntas Infectologia
Page 131: Preguntas Infectologia
Page 132: Preguntas Infectologia

ENFERMEDAD POR RASGUNO DE GATO

• Cual es el agente causal de la enfermedad por rasguño de gato?• R = BARTONELLA HENSELEAE

• -A 30-year-old man develops a pustular lesion at the site of a cat scratch on his forearm. This is followed 1 week later by malaise, fever, and lymphadenopathy. On examination, he has tender axillary lymph nodes. Which of the following is the most likely causative organism?

• (A) Bartonella henselae• (B) Bartonella bacilliformis• (C) Bartonella quintana• (D) Coxiella burnetii• (E) Borrelia burgdorferi

Page 133: Preguntas Infectologia

SARAMPION

• Cuales son los signos y síntomas de Sarampión?• R = TOS, CORIZA Y CONJUNTIVITIS CON FOTOFOBIA. MANCHAS DE

KOPLIC (Lesión blanquecina sobre base eritematosa en mucosa subyugal). Exantema que inicia cabeza, progresión céfalo-caudal RESPETA PALMAS Y PLANTAS.

Page 134: Preguntas Infectologia

CASO CLINICO-A 7-year-old child, unvaccinated because of his parents’ religious beliefs,

develops malaise, cough, coryza, and conjunctivitis with a high fever. Examination of his mouth reveals blue white spots on a red base beside his second molars. The next day he develops an erythematous, nonpruritic, maculopapular rash at his hairline and behind his ears, which spreads over his body. Which of the following is the most likely diagnosis?

• (A) hand-foot-and-mouth disease (coxsackievirus)• (B) measles (rubeola)• (C) rubella (German measles)• (D) mumps• (E) pertussis

Page 135: Preguntas Infectologia

- This is a typical case of measles. The Koplik’s spots in the mouth are easily missed with poor illumination. They are white blue spots of 1 mm on a red background and are not seen in any other infectious disease. The rash of measles becomes confluent while that of rubella does not. Pneumonia is an infrequent complication but accounts for many measles deaths. Giant cell pneumonia is also seen, most commonly in children suffering with a severe disease such as leukemia or immunodeficiency. Aerosolized ribavirin has been used to treat severe pneumonia secondary to measles, but its efficacy is still unclear. The other potentially lethal complication of measles is encephalitis

Page 136: Preguntas Infectologia

CASO CLINICO

- An 8-year-old boy from an impoverished innercity area has never been vaccinated appropriately. He develops fever, cough, and coryza. The next day, blue white spots develop on the buccal mucosa. On the third day, an erythematous, nonpruritic maculopapular rash develops on the face and spreads over the entire body. Which of the following is the most likely complication?

• (A) pneumonia• (B) encephalitis• (C) otitis media• (D) bronchitis• (E) mastoiditis

Page 137: Preguntas Infectologia

- The most common complication of measles is otitis media, other complications include mastoiditis, pneumonia, bronchitis, encephalitis, and lymphadenitis. Otitis media is usually a bacterial superinfection, and should be treated with antibiotics.

Page 138: Preguntas Infectologia

CASO CLINICO- A6-year-old boy develops symptoms of cough, fever, and malaise followed by

a generalized maculopapular rash that has spread from the head downwards. A clinical diagnosis of measles is made. A few days after the onset of the rash he is drowsy, lethargic, and complaining of headache. A lumbar puncture, electroencephalogram (EEG), and computerized tomography (CT) of the brain exclude other etiologies and confirm the diagnosis of encephalitis. Which of the following is the most likely delayed neurologic complication of measles virus encephalitis?

• (A) meningitis• (B) pure motor paralysis• (C) autonomic neuropathy• (D) mental retardation or epilepsy• (E) “stocking-glove” peripheral neuropathy

Page 139: Preguntas Infectologia

- Subacute sclerosing panencephalitis causes involuntary spasmodic movements and progressive mental deterioration, frequently ending in death within a year. It usually occurs in children whose measles occurred at an early age (= 2 years). It occurs 6–8 years after the primary infection. It presents with nonspecific symptoms such as poor school performance or mood and personality changes. It then progresses to intellectual decline, seizures, myoclonus, ataxia, and visual disturbances. Continued deterioration results in inevitable death.

Page 140: Preguntas Infectologia

MONONUCLEOSIS• Cual es el cuadro clínico de un paciente con CMV/Mononucleosis?• R = Faringo- amigdalitis, ADENOPATÍAS CERVICALES- AXILARES ESPLENOMEGALIA.

• - A 21-year-old woman visits her physician because of 3 weeks of a “flu-like” illness. She reports symptoms of malaise, fever, fatigue, and a sore throat. There is no weight loss or night sweats, and she has not traveled out of country. Her past medical history is not significant and she is not taking any medications. Physical examination is normal except for enlarged cervicallymph nodes. Laboratory data show hemoglobin 13.2 g/dL; hematocrit 42%; platelets 380,000/mL; WBC 8500/mL, with 35% segmented neutrophils, 1% eosinophils, and 64% lymphocytes, of which 36% were atypical. A heterophil antibody (sheep cell agglutination) test is negative. Which of the following is the most likely causative organism?

• (A) herpes simplex• (B) echovirus• (C) CMV• (D) coxsackievirus• (E) reovirus

Page 141: Preguntas Infectologia

-The most common cause of non-EBV mononucleosis-type syndrome is CMV. It is the most common presentation of CMV in nonneonates with normal immune function.

Page 142: Preguntas Infectologia

MONONUCLEOSIS INFECCIOSA O ENFERMEDAD DEL BESO

• Cual es el agente causal de la mononucleosis infecciosa o enfermedad del beso?• R = EPSTEIN BARR y es un herpes virus

• A que patologías se asocia el virus del Epstein Bar?• R = LINFOMA DE BURKITT, linfoma de células T, LEUCOPLASIA ORAL VELLOSA,

carcinoma nasofaríngeo.

• Como dx al virus de Epstein bar?• R = AGLUTININAS HETEROFILICAS, MONO test.

• Cual es el tx de Epstein Bar?• R = ACICLOVIR

Page 143: Preguntas Infectologia

CASO CLINICO• -A 17-year-old man presents with new symptoms of fatigue, malaise, fever, and a

sore throat. He has no significant past medical history and is not on any medications. Physical examination is entirely normal except for enlarged, palpable cervical, lymph nodes. He reports no weight loss or night sweats. Laboratory investigations include a normal chest x-ray, negative throat swab, but abnormal blood film with atypical lymphocytes. The hemoglobin is 15.5 g/dL; hematocrit 42%; platelets 290,000/mL; WBC 10500/mL, with 45% segmented neutrophils, 1% eosinophils, and 54% lymphocytes, of which 36% were atypical. Which of the following is the most appropriate initial diagnostic test?

• (A) lymph node biopsy• (B) bone marrow• (C) erythrocyte sedimentation rate (ESR)• (D) heterophil antibody (sheep cell agglutination) test• (E) hepatic biopsy

Page 144: Preguntas Infectologia

-The presence of IgG antibodies by the indirect immunofluorescence test indicates recent or prior EBV infection. IgM antibodies indicate recent infection only. Heterophil antibodies are present in 50% of children and 90–95% of adolescents and adults with infectious mononucleosis. Monospot tests are the best diagnostic tools but may not turn positive until the second or third week of the illness. Specific EBV antibodies and cultures are rarely used.

Page 145: Preguntas Infectologia
Page 146: Preguntas Infectologia

HERPEANGINA

• Que enfermedades causa el VIRUS COXSACKIE?• Grupo A : HERPEANGINA, ENFERMEDAD MANO-PIE-BOCA y

conjuntivitis hemorrágica aguda, el • Grupo B:pleurodinia, miocarditis, pericarditis y

meningoencefalitis.

• Cuales son las características de la herpeangina?• R = LESIONES VESICULOSAS Y ULCEROSAS EN FARINGE,

PARTICULARMENTE EN LOS PILARES ANTERIORES, las amígdalas y el paladar, vomito, anorexia, fatiga y ataque al estado general.

Page 147: Preguntas Infectologia

CASO CLINICO

- A previously healthy 19-year-old female university student develops myalgia, headache, fever, and malaise. Blood tests reveal lymphocytosis, with 20% of the lymphocytes being atypical. She remains tired and unwell for 6 weeks, but repeated tests for heterophil antibody are negative. Which of the following is the most likely diagnosis?

• (A) Epstein-Barr virus (EBV) infection• (B) primary HIV infection• (C) human herpes virus type 7 (HHV-7)• (D) CMV infection• (E) toxoplasmosis

Page 148: Preguntas Infectologia

VALVULAS NATIVAS• Cual es el agente causal de endocarditis en caso de reemplazo valvular con válvulas

nativas?• R = STAPHYLOCOCCUS EPIDERMIDIS

- 60-year-old man presents with fever and malaise 6 weeks after mitral valve replacement. On examination, his temperature is 38C, blood pressure 130/80 mm Hg, pulse 80/min, and a loud pansystolic murmur at the apex, which radiates to the axilla. He has no skin or neurologic findings. Which of the following is the most likely causative organism?

• (A) Staphylococcus aureus• (B) a fungus• (C) Staphylococcus saprophyticus• (D) pneumococcus• (E) Staphylococcus epidermidis

Page 149: Preguntas Infectologia

- About half of all early-onset (<60 days after surgery) prosthetic endocarditis is caused by staphylococcal infection, with S. epidermidis predominating. Early-onset prosthetic endocarditis is generally the result of intraoperative contamination of the prosthesis or a bacteremic postoperative complication.

Page 150: Preguntas Infectologia

CASO CLINICO- A 56-year-old man is having intermittent fevers and malaise for the past 2 weeks.

He has no other localizing symptoms. Two months ago, he had valve replacement surgery for a bicuspid aortic valve. Amechanical valve was inserted and his postoperative course was uncomplicated. On examination, his temperature is 38C, blood pressure 124/80 mm Hg, pulse 72/min, and head and neck are normal. There is a 3/6 systolic ejection murmur, the second heart sound is mechanical, and a 2/6 early diastolic murmur is heard. The lungs are clear and the skin examination is normal. Three sets of blood cultures are drawn and an urgent echocardiogram is ordered. Which of the following is the most likely causative organism?

• (A) Staphylococcus aureus• (B) S. epidermidis• (C) S. viridans• (D) enterococci• (E) Candida

Page 151: Preguntas Infectologia

- S. epidermidis is still the leading cause of prosthetic valve endocarditis in the early postoperative period (usually defined <2 months). It is usually the consequence of intraoperative contamination or postoperative bacteremia. Endocarditis occurring 12 months after surgery is usually due to the same organisms that cause native valve endocarditis.

Page 152: Preguntas Infectologia

ENDOCARDITIS• Cuales son los agentes infecciosos involucrados en la endocarditis sub aguda?• R = Estreptococos del grupo viridans

• Cual es el tratamiento para los estreptococos del grupo viridans?• R = PENICILINA + AMINOGLUCOSIDO POR 30 DÍAS

• Cual es el agente causal de endocarditis de Litman Sax?• R = AUTOINMUNE, común en LUPUS.

• Cual es el procedimiento terapéutico profiláctico para intervención cardiaca o dental?

• R = AMOXICILINA 2 GR 1 HR ANTES DEL PROCEDIMIENTO, si es ALÉRGICO

CLINDAMICINA 600 MG, AZITROMICINA 500 MG 1 HR ANTES DEL EVENTO.

• En endocarditis meticilino resistente que medicamento das?

• R = TEICOPLANINA, en gram (-) cefalosporinas de 3era

Page 153: Preguntas Infectologia
Page 154: Preguntas Infectologia
Page 155: Preguntas Infectologia
Page 156: Preguntas Infectologia

NEUMONIA• Cual es el principal agente patógeno en neumonía adquirida en la comunidad?• R = Streptococcus pneumoniae

• -A 73-year-old man from a nursing home develops headache, fever, cough, sore throat, malaise, and severe myalgia during a community outbreak affecting numerous other residents at the home. The symptoms gradually resolve after 3 days, and he starts feeling better but then there is a reappearance of his fever,with cough and yellow sputum production. On examination, his temperature is 38.5C, pulse 100/min, respiration 24/min, oxygen saturation 88% and crackles in the right lower lung base, bronchial breath sounds and dullness on percussion. CXR reveals a new infiltrate in the right lower lobe. Which of the following is the most likely causative organism?

• (A) primary viral pneumonia• (B) an autoimmune reaction• (C) Mycoplasma pneumoniae• (D) Streptococcus pneumoniae• (E) Neisseria catarrhalis

Page 157: Preguntas Infectologia

- This man has a complication of viral influenza; a secondary bacterial pneumonia has developed. This usually occurs 2–3 days after the initial viral symptoms resolve. Primary viral pneumonia with influenza is not common. S. pneumoniae, Staphylococcus, and H. influenzae are the most common bacterial invaders in pulmonary complications of influenza. Pneumonia is the leading cause of death and may also be due to S. pneumonia and H. influenzae. Mixed viral and bacterial pneumonia is common; pure viral pneumonia in influenza is uncommon (but can be very severe).

Page 158: Preguntas Infectologia

NEUMONIA• Cual es el patógeno mas frecuente en la comunidad?• 1. Streptococo pneumonie 2.Micoplasma pneumonie 3.Clamidia pneumoniae

• Agente etiológico de neumonía hospitalaria?• 1.Pseudomona 2. Kliebsella 3.E.Coli

• Agentes etiológicos de neumonía adquirida en la comunidad con bronco aspiración?

• Px ALCOHÓLICOS y/ o PX QUE CONVULSIONAN y los agentes BACTEROIDES ORALIS, streptococo

• Agentes etiológicos para neumonía en px atípica?• INFLUENZA H1N1 MICOPLASMA PNEUMONIE

• Caso clínico: paciente que estuvo en contacto con aire acondicionados o torre de enfriamento o nebulazadores que durante un tiempo no funciono… desarrolla neumonía

• cual seria el agente causal?• Legionella pneumonie

• Tx:

• ERITROMICINA o CLARITROMICINA si viene

Page 159: Preguntas Infectologia

NEUMONIA• Enfermedad ocupacional que desarrollan personas que cuidan pájaros?

• PSITACOSIS agente causal: CHLAMYDIA PSITTACI • Tx :

• TETRACICLINA (doxiciclina)

• Caso clínico de MICOPLASMA PNEUMONIE caract y tx?• Es una neumonía atípica FENOMENO DE REYNAUD, ERITEMA MULTIFORME. Hemolisis, MIRINGITIS BULOSA Rx:

Pneumoniae en Parches

• Tx:

• ERITROMICINA (si vienen azitromicina o claritromicina contestar esas)

• Forma en que se realiza el dx:

• Prueba de fijación de complementos (la de mas especificidad y sensibilidad )ANTICUERPOS FRÍOS

• En que consiste el sx de austria?• ENDOCARDITIS, NEUMONÍA y MENINGITIS…. Se presenta en px ALCOHÓLICOS • Agente causal: • STRPTOCOCO PNEUMONIE

• Cual es la imagen radiográfica de la neumonía adquirida en la comunidad: Broncograma aéreos dentro de la consolidación pulmonar.

Page 160: Preguntas Infectologia

NEUMONIA• Que neumonía se relaciona con neumatocele?• Neumonía estafilococica (burbujas de aire en el parénquima pulmonar)?

• La neumonía adquirida en el hospital cual es su imagen radiológica?• Patrón necrotizante, de focos múltiples

• Neumonía adquirida en comunidad por bronco aspiración con pérdida del conocimiento que partes está afectada?

• Esta afectado el PULMÓN DERECHO, el LÓBULO MEDIO y el SEGMENTO POSTERIOR

• Neumonia en un px con vih cual es su agente causal?• Neumonía por PNEUMOCYSTIS JIROVECI

Page 161: Preguntas Infectologia

NEUMONIA• Px con VIH con sospecha de neumonía por pneumocystis jiroveci que tinción se realiza para corraborar dx?• R: metenamina argéntica por NITRATO DE PLATA

• Px con micoplasma el mejor estudio es?• 1era pruebas de FIJACIÓN DE COMPLETOS • 2do opción ANTICUERPOS FRÍOS

• Tratamiento de neumonía adquirida en la comunidad?1) Si no tiene enfermedad concomitante: 2) MACROLIDO (azitromicina o claritromicina) tx de 1era elección3) Con enfermedad concomitante: 4) FLUOROQUINOLONAS (levo, moxi, gatifloxacina)

• Tx de px con neumonía adquirida en la comunidad pero que por sus caract. Es internado?• CEFALOSPORINA DE 3RA GENERACIÓN MAS MACROLIDO: ej cefotaxima + claritromicina

• Tx de neumonía por neumococo?

• FLUOROQUINOLONA LEVOFLOXACINO (NO poner PENICILINA… ojo)

Page 162: Preguntas Infectologia
Page 163: Preguntas Infectologia

NEUMONIA

• Px con neumonía adquirida en comunidad pero VIENE GRAVE que debe ser INTERNADO EN TERAPIA intensiva?

• Fluoroquinolona + ceftriaxona o fluoroquinolona + cefotaxima

• Tx paciente con neumonía ADQUIRIDA INTRAHOSPITALARIA, internado en terapia intensiva?

• Pseudomona, ecoli, kliebsella FLUOROQUINOLONA + CEFALOSPORINA antipseudomonica (CEFTAZIDIMA, cefoperazona) o (carbepenem o iminpenem ) solo

• Neumonía hospitalaria por staphilococos aureus cual es el tx?• VANCOMICINA (útil incluso para meticilino resistentes)

Page 164: Preguntas Infectologia
Page 165: Preguntas Infectologia
Page 166: Preguntas Infectologia
Page 167: Preguntas Infectologia
Page 168: Preguntas Infectologia

CASO CLINICO

- A 4-year-old boy is sent to the emergency room because of clinical suspicion of meningitis. He has been ill for 2 days with fever and lethargy. On examination, he is febrile, the neck is stiff, and papilledema is present. There is no rash, the lungs are clear, and heart sounds normal. Which of the following is the most likely causative organism?

• (A) Neisseria meningitidis• (B) Streptococcus pneumoniae• (C) Haemophilus influenzae• (D) Staphylococcus• (E) Listeria species

Page 169: Preguntas Infectologia

- Since the introduction of H. influenzae type B vaccine, S. pneumoniae has become the most common type of meningitis in infants and toddlers

Page 170: Preguntas Infectologia

TUBERCULOSIS• Cuales son las manifestaciones extrapulmonares de la Tb?• R = Meningitis, pericarditis, invasión ósea, gastrointestinal y urinaria

• Cual es la tinción especifica para Tb y reacción PPD positiva?• R = Tinción de ZIEHL NEELSEN. En paciente inmunocomprometido ES + SI ES > 5 MM

• Como se muestra el LCR con Tb?• R = CON PROTEÍNAS Y CELULARIDAD AUMENTADAS E HIPOGLUCORRAQUIA.

• Como se da el tratamiento para Tb?• FASE INTENSIVA: Se dan 3 FÁRMACOS MAS ETAMBUTOL Y ESTREPTOMICINA hasta completar 60 DOSIS• FASE DE SOSTEN: ISONIAZIDA Y PIRAZINAMIDA 3 VECES A LA SEMANA DANDO 45 DOSIS.

• Como das quimioprofilaxis a contactos de pacientes con Tb?• R = 6M EN CONTACTOS <5ª Y EN LOS CONTACTOS DE 5 – 14ª NO VACUNADOS CON ISONIAZIDA SIN

EXCEDER 300 MG.

• Como se le llama a la disminucion de la respuesta tuberculinica con el paso de la edad?• R = Efecto Spink

Page 171: Preguntas Infectologia

TUBERCULOSIS• Cual es el mecanismo de acción de la RIFAMPICINA?• R = Es un BACTERICIDA que inhibe la subunidad beta de la RNA POLIMERASA,

IMPIDIENDO LA SÍNTESIS PROTEÍNICA.

• Cual es el mecanismo de acción de la ISONIACIDA?• R = Es un ANÁLOGO DE LA NICOTINAMIDA que inhibe la SÍNTESIS DE ACIDO MICOLICO y

el metabolismo de los lípidos que CAUSA LA ROTURA DE LA PARED, el cual DEBE

ADMINISTRARSE CON PIRIDOXINA YA QUE PUEDE CAUSAR NEUROPATÍA PERIFÉRICA. Se usa la Estreptomicina en caso de resistencia a Isoniacida.

• Cual es el mecanismo de acción de la PIRAZINAMIDA?• R = ANALOGO DE LA NICOTINAMIDA que actúa como BACTERICIDA en un medio acido,

lo que le permite actuar contra los bacilos intracelulares.

• Cual es el mecanismo de acción del ETAMBUTOL?• R = Tiene actividad BACTERIOSTÁTICA. En menores de 12ª puede causar NEURITIS

RETROBULBAR.

Page 172: Preguntas Infectologia

TUBERCULOSIS• Grupo de edad en que es mas frecuente la micobacteriosis ganglionar?• Es mas frecuente en los niños (en segundo lugar la tb pulmonar) se divide en escrofulacia y la

tuberculosa

• Grupo de edad en que es mas frecuente la micobacteriosis pulmonar?• En adultos: la primera causa es la tb pulmonar y la #2 la tb ganglionar

• Causa mas frecuente de Síndrome de piura estéril? • 1era. Tuberculosis Renal • 2da. Micoplasma Ominis

• Características de TB Renal?1) Dolor lumbar, disuria, Febricula, 2) Urografía excretor: URETEROS ARROSARIADOS

• Cuando afecta ala columna vertebral se llama? • Mal de Pott

• Cuanto tiempo se debe mantener en aislamiento a alguien que padece tuberculosis activa?• R = 2-3 meses

Page 173: Preguntas Infectologia

TUBERCULOSIS• Diagnostico diferencia de tb intestinal? • Enfermedad de Cronh y cáncer intestinal

• Cual es la utilidad de la BCG?• Proteger contra MENINGITIS TUBERCULOSA en los niños (su protección no dura mas de

10 años)

• Características de una meningitis tuberculosa?• Evolución subaguda, tiende a obstruir los agujeros de lushka y maggendi,

CARACTERÍSTICAMENTE EL 3-4-6 PAR CRANEAL AFECTADO

• Cuales son las dos micobacteriosis, en los cuales en el esquema de tratamiento se agrega corticoesteroides?

• Meningoencefalitis tuberculosa y pericarditis tuberculosa

• Causa mas frecuente de enfermedad de adison?• 1era.AUTOINMUNIDAD 2da:TUBERCULOSIS (mas frecuente en países

subdesarrollados)

Page 174: Preguntas Infectologia

TUBERCULOSIS• Cual es la causa mas frecuente de origen infeccioso de calcificación de suprarrenales?• Enfermedad de Adison (astenia, adinamia, HIPERPIGMENTACION CUTANEA elevada,

NA BAJO, K ELEVADO, acidosis elevada, hipotensial arterial)

• Diagnostico definitivo de TB?• Cultivo LOWENSTEIN-JENSTEIN-HOLTZ

• Que es el complejo de Ranke?• Nódulos linfático hiliares o parahiliares afectados por tb + complejo primario de Gonh

• Cual es el sitio mas frecunte del complejo primario de Gonh?• Lóbulo inferior del lóbulo derecho

• Como se interpreta Intradermoreaccion de Monteaux?• Se lee a las 72 HRS DE HABERLO APLICADO.. SI TIENE MAS DE (10 MM) 1 CM DE

INDURACIÓN SE CONSIDERA POSITIVO

Page 175: Preguntas Infectologia

TUBERCULOSIS• Si es un px con VIH, a cuanto se considera positiva? • .5 cm (5MM)

• Que medicamentos constituye los 4 meses restantes de tx de px con tb? • Isoniacida + rifampicina

• Que antifimico queda contraindicado en la mujer embarazada?• ESTREPTOMICINA…. El tx es con ISO RIFA ETAMBUTOL + PIRIDOXINA POR 9 MESES (para

disminuir el efecto indeseable de la isoniacida –polineuropatia y hepatitis-)

• Cuales son las tuberculosis que se da tx por 9 meses? • En la tb MILIAR, tb SNC, tb ARTICULAR u OSEA

• Los px con resistencia solo a isoniacida que tx se deberá dar? (Se llama RESISTENTE si lo es a isoniacida)

1) RIFAMPICINA, PIRASINAMIDA Y ETAMBUTOL O RIFAMPICINA, PIRASINAMIDA ESTREPTOMICINA POR 6 MESES

2) Rifampicina + etambutol por un año (se llama MULTIRESISTENTE si es a iso y rifampicina)

Page 176: Preguntas Infectologia

TUBERCULOSIS• Dosis de la isoniacida? • 5mg por kilo de peso

• Dosis de la rifampicina? • 10 mg x kg de peso

• Dosis de etambutol? • 15mg x kg de peso

• Dosis de la pirasinamida en la tb?• Menos de 50 kg: 1 gr por dia, entre 50 y 70:1.5 gr x dia mas de 70: mas de 2 gr x dia

• Efecto indeseable mas importante en rifampicina? • Hepatotoxicidad y nefrotoxicidad

• Antifimico que tiñe de color naranja los líquidos corporales? • Rifampicina

• Efecto indeseable del etambutol? • Neuritis óptica

• Antifimico que es bacteriostático? • Etambutol

• Efecto indeseable de la pirasinamida? • Hepatotoxicidad e hiperuricemia

• Mecanismo de acción de la isoniacida? • Actúa inhibiendo el acido mucolico de la micobacteria

• Mecanismo de acción de la rifampicina? • Inhibe la girasa del dna

Page 177: Preguntas Infectologia
Page 178: Preguntas Infectologia
Page 179: Preguntas Infectologia
Page 180: Preguntas Infectologia
Page 181: Preguntas Infectologia
Page 182: Preguntas Infectologia

ENTEROVIRUS• Cual es el principal agente causal de meningitis viral en niños durante el verano?• R = Enterovirus

• - A 10-year-old boy presents with fever, headache, photophobia, and neck discomfort in the middle of summer. He is alert and oriented, but has neck pain with flexion and extension of the head. His fundi are normal, and there are no focal neurologic findings or skin changes. A lumbar puncture reveals normal protein and glucose with a cell count of 240/mL (90% lymphocytes). Which of the following is the most likely causative organism?

• (A) enterovirus (coxsackievirus or echovirus)• (B) Streptococcus pneumoniae• (C) Neisseria meningitides• (D) Listeria monocytogenes• (E) HSV-1

Page 183: Preguntas Infectologia

- The cerebrospinal fluid (CSF) picture in this individual is consistent with viral meningitis rather than a bacterial process. Enteroviruses are a prominent cause of viral meningitis in the summer and fall months. They received their name because they multiply in the GI tract. Fever, sometimes associated with respiratory symptoms, is the most common sequel of enterovirus infection. There are about 70 enteroviruses that affect humans. These include polioviruses, coxsackieviruses, echoviruses, and others. The spectrum of disease includes paralytic disease, encephalitis, aseptic meningitis, pleurodynia, exanthems, pericarditis, myocarditis, and nonspecific febrile illnesses. They can on occasion cause fulminant disease in a newborn. The most important enteroviruses are the three poliovirus serotypes.

Page 184: Preguntas Infectologia

DIARREA DEL VIAJERO• Sx diarreico agudo relacionado con la ingesta de arroz?• R = BACILUS CEREUS

• Tratamiento actual para diarrea del viajero?• R = CIPROFLOXACINO

-A 34-year-old man is traveling in Southeast Asia on business. He is staying in Western-style hotels and eating food in large restaurants. He has not eaten from street vendors. One week after arrival, he develops symptoms of anorexia, nausea, and abdominal cramps followed by the sudden onset of watery diarrhea. He has no fever or chills and there is no blood or pus in the stools. Which of the following is the most appropriate therapy for his condition?

• (A) amoxicillin• (B) symptomatic therapy with loperamide• (C) doxycycline• (D) oral rehydration only• (E) specific antitoxin

Page 185: Preguntas Infectologia

-Current recommendations suggest that mild diarrhea be treated with oral rehydration alone, but when enteric symptoms such as cramps are bothersome, treatment with loperamide or bismuth subsalicylate is warranted. More severe infections with severe diarrhea, severe pain, or fever should be treated with antibiotics such as fluoroquinolones or trimethoprim-sulfa

Page 186: Preguntas Infectologia
Page 187: Preguntas Infectologia
Page 188: Preguntas Infectologia

INMUNODEFICIENCIAS

Page 189: Preguntas Infectologia

• 4

Page 190: Preguntas Infectologia
Page 191: Preguntas Infectologia

HANTAVIRUS• Como se contrae el hantavirus?• El hantavirus es un virus perteneciente a la FAMILIA BUNYAVIRIDAE. La infección por hantavirus

es causada cuando una persona entra en contacto con roedores que están infectados con hantavirus, o cuando una persona entra en contacto con orina o excrementos de roedores infectados. Aproximadamente, del 30% al 40% de personas que contraen infección por hantavirus morirán. La infección por hantavirus no se puede transmitir entre humanos.

• Cuáles son los síntomas de la enfermedad por hantavirus? • Los síntomas iniciales son dolores musculares y fiebre (101-104F). Otros síntomas comunes son

dolor de cabeza, tos, náusea ó vómito, diarrea, y dolor abdominal. Sin embargo, EL SÍNTOMA PRIMARIO DE LA ENFERMEDAD DEL HANTAVIRUS ES LA DIFICULTAD DE RESPIRAR QUE ES CAUSADA POR LA ACUMULACIÓN DE FLUIDOS EN LOS PULMONES. Esto puede ocasionar paro respiratorio ó inhabilidad de respirar. Típicamente, estos problemas respiratorios se desarrollan algunos días después de los síntomas iniciales. En algunos casos de la enfermedad por hantavirus, los riñones y otros órganos paran de trabajar.

• Cuál es el tratamiento para la enfermedad por hantavirus? • Actualmente, NO HAY TRATAMIENTO ESPECÍFICO PARA LA ENFERMEDAD por hantavirus. El

cuidado intensivo temprano en hospital es actualmente la única medida conocida.

Page 192: Preguntas Infectologia

MICELANEAS• Cuales son las afirmaciones sobre la profilaxis antimicrobiana en cirugía?1) Una dosis inmediatamente antes de la cirugía proporciona concentraciones tisulares adecuadas durante toda la

intervención2) Si se prolonga la cirugía se aconseja una segunda dosis durante el procedimiento3) El objetivo de la profilaxis debe ser reducir la cantidad de gérmenes por debajo del nivel critico necesario para

producir infección4) La cefazolina es el fármaco de elección en la mayoría de los casos

• Que forma de sífilis tratarías con Penicilina G acuosa?• R = Sífilis congénita

• Cual es el agente causal de la ornitosis?• R = Chlamydia psittaci. Es sinonimo de Psitacosis.

• Cual es el microorganismo que con mayor frecuencia da otitis media aguda?• R = 1) Neumococo (Streptococo neumoniae) 2) Hemophilus Influenzae y 3) Moraxela C.

• Cuales son las complicaciones de la otitis media supurada?• R = Mastoiditis, Hipoacusia, Laberintitis y parálisis facial.

• Cual es la forma de candidiasis oral mas común?• R = Estomatitis aguda pseudomembranosa

Page 193: Preguntas Infectologia

• Cual es el tratamiento de elección en la celulitis periorbitaria?• R = Amoxicilina

• Cual es el agente causal de la gangrena gaseosa?• R = Clostridium perfringes 80%

• Que antimicrobiano cubre casi todos los agentes patógenos de una mordedura por animal o persona?

• R = Amoxi- Acido clavulanico

• Cual es el agente bacteriano en la mordida por ser humano?• R = EIKENELLA CORRODENS

• Cual es el manejo de la toxoplasmosis cerebral?• R = Primetamina + sulfadiazina + acido fólico

• Cual es el método diagnostico de laboratorio para Ricketsia?• R = Proteus OX-19 1:320 + clínica, con mucha fiebre, cefalea y rash cutáneo.

• Como se le llama a la enfermedad contraída por aves en general?• R = Psitacosis, por clamidia psitasi

• Cuales son los agentes etiológicos implicados en el síndrome de guillain-barre?• R = Campylobacter jejuni y CMV

Page 194: Preguntas Infectologia

Consejos

1. Leer diariamente y en bloques

2. Adiós Partys un tiempo

3. Has ejercicio y come bien durante el estudio

4. Toma algún curso bueno si tienes la posibilidad

5. Ten Fe.

Page 195: Preguntas Infectologia

BIBLIOGRAFIA

• EXARMED• PAPADAKIS• CTO• HARRISON• AMIR• USMLE STEPS